The words you are searching are inside this book. To get more targeted content, please make full-text search by clicking here.

Rapid Review of Clinical Medicine for MRCP Part 2 by Sharma, Sanjay Kaushal, Rashmi (z-lib.org)

Discover the best professional documents and content resources in AnyFlip Document Base.
Search
Published by srini vasan, 2020-04-28 03:03:52

Rapid Review of Clinical Medicine for MRCP Part 2 by Sharma, Sanjay Kaushal, Rashmi (z-lib.org)

Rapid Review of Clinical Medicine for MRCP Part 2 by Sharma, Sanjay Kaushal, Rashmi (z-lib.org)

Rapid Review of

CLINICAL
MEDICINE

for MRCP Part 2

Second Edition

Sanjay Sharma • Rashmi Kaushal

MANSON BCeasstesse,ll•aenrRs–ewflefeurcslt,lsyenxrepewvlaisneeaxdtaim&onfuso,prtdmuatatoetrdials

PUBLISHING






Rapid Review of

Clinical
Medicine

for MRCP Part 2

Second Edition

Sanjay Sharma

BSc (Hons) MD FRCP (UK) FESC
Professor of Clinical Cardiology

Consultant Cardiologist and Physician
St George’s University of London
St George’s Hospital NHS Trust
University Hospital Lewisham
London, UK

Rashmi Kaushal

BSc (Hons) FRCP (UK)
Consultant Physician and Endocrinologist

West Middlesex Hospital
Kingston, UK

MANSON

PUBLISHING

CRC Press
Taylor & Francis Group
6000 Broken Sound Parkway NW, Suite 300
Boca Raton, FL 33487-2742
© 2006 by Taylor & Francis Group, LLC
CRC Press is an imprint of Taylor & Francis Group, an Informa business

No claim to original U.S. Government works
Version Date: 20150311

International Standard Book Number-13: 978-1-84076-641-7 (eBook - PDF)

This book contains information obtained from authentic and highly regarded sources. While all reasonable efforts have been made to publish reliable
data and information, neither the author[s] nor the publisher can accept any legal responsibility or liability for any errors or omissions that may be
made. The publishers wish to make clear that any views or opinions expressed in this book by individual editors, authors or contributors are personal
to them and do not necessarily reflect the views/opinions of the publishers. The information or guidance contained in this book is intended for use
by medical, scientific or health-care professionals and is provided strictly as a supplement to the medical or other professional’s own judgement, their
knowledge of the patient’s medical history, relevant manufacturer’s instructions and the appropriate best practice guidelines. Because of the rapid
advances in medical science, any information or advice on dosages, procedures or diagnoses should be independently verified. The reader is strongly
urged to consult the relevant national drug formulary and the drug companies’ and device or material manufacturers’ printed instructions, and their
websites, before administering or utilizing any of the drugs, devices or materials mentioned in this book. This book does not indicate whether a par-
ticular treatment is appropriate or suitable for a particular individual. Ultimately it is the sole responsibility of the medical professional to make his or
her own professional judgements, so as to advise and treat patients appropriately. The authors and publishers have also attempted to trace the copyright
holders of all material reproduced in this publication and apologize to copyright holders if permission to publish in this form has not been obtained. If
any copyright material has not been acknowledged please write and let us know so we may rectify in any future reprint.

Except as permitted under U.S. Copyright Law, no part of this book may be reprinted, reproduced, transmitted, or utilized in any form by any elec-
tronic, mechanical, or other means, now known or hereafter invented, including photocopying, microfilming, and recording, or in any information
storage or retrieval system, without written permission from the publishers.

For permission to photocopy or use material electronically from this work, please access www.copyright.com (http://www.copyright.com/) or contact
the Copyright Clearance Center, Inc. (CCC), 222 Rosewood Drive, Danvers, MA 01923, 978-750-8400. CCC is a not-for-profit organization that pro-
vides licenses and registration for a variety of users. For organizations that have been granted a photocopy license by the CCC, a separate system of
payment has been arranged.

Trademark Notice: Product or corporate names may be trademarks or registered trademarks, and are used only for identification and explanation
without intent to infringe.
Visit the Taylor & Francis Web site at
http://www.taylorandfrancis.com
and the CRC Press Web site at
http://www.crcpress.com

Contents 3

Acknowledgements 2
Preface 3
Classification of Cases 4
Abbreviations 5
Clinical Cases 7
Data Interpretations Tutorials 415
415
Calcium Biochemistry 415
Genetics 416
Audiograms 418
Guidelines for the Interpretation of Cardiac Catheter Data 419
Respiratory Function Tests 420
Interpretation of Respiratory Flow Loop Curves 421
Echocardiography 426
Acid–base Disturbance 427
Normal Ranges

Preface

Passing specialist examinations in internal medicine is a diagnoses, diagnostic algorithms and up-to-date medical
difficult milestone for many doctors, but is a mandatory lists are presented. Many questions comprise illustrated
requirement for career progression. Pass rates in these material in the form of radiographic material, electro-
examinations are generally low due to ‘high standards’ cardiograms, echocardiograms, blood films, audiograms,
and ‘stiff competition’. Thorough preparation is essential respiratory flow loops, histological material, and slides in
and requires a broad knowledge of internal medicine. ophthalmology, dermatology and infectious diseases.
The pressures of a busy clinical job and nights ‘on call’ Over 200 commonly examined illustrations are included.
make it almost impossible for doctors to wade through
heaps of large text books to acquire all the knowledge Tutorials are included at the end of the book to aid
that is required to pass the examinations. the interpretation of illustrated material as well as impor-
tant, and sometimes difficult, clinical data, such as respir-
The aim of this book is to provide the busy doctor with atory function tests, cardiac catheter data and dynamic
a comprehensive review of questions featured most endocrine tests.
frequently in the MRCP (II) examination in internal
medicine. The MRCP (II) examination has a best of 5/n The book will prove invaluable to all those studying
from many answer format. The vast majority of the for higher examinations in internal medicine, and to their
questions in the book follow the same pattern; however, instructors.
we have chosen to include several scenarios with open
ended questions to stimulate the medical thought process. Sanjay Sharma
The level of difficulty of each question is of the same Professor of Clinical Cardiology
standard as MRCP (II) examination. However, some cases Consultant Cardiologist and Physician
are deliberately more difficult for teaching purposes. Lecturer for Medibyte Intensive Courses

A broad range of subjects is covered in over 400 for the MRCP Part 2
questions ranging from metabolic medicine to infectious
diseases. Precise answers and detailed discussion follow Rashmi Kaushal
each question. Where appropriate, important differential Consultant Physician and Endocrinologist

4

Classification of Cases

Cardiology Metabolic medicine
1, 10, 11, 13, 22, 25, 32, 40, 52, 53, 54, 62, 63, 66, 68, 9, 29, 34, 38, 50, 71, 74, 81, 82, 84, 90, 129, 134, 136,
74, 78, 80, 94, 95, 100, 121, 123, 125, 130–132, 138, 147, 153, 161, 179, 189, 214, 215, 230, 248, 257, 271,
144, 150, 160, 167, 178, 180, 184, 193, 197, 199, 202, 275, 283, 310, 321, 326, 329, 333, 334, 398
203, 207, 208, 223, 226, 229, 232, 235, 237, 243, 246,
259, 266, 270, 285, 287, 291, 296, 301, 305, 307, 309, Nephrology
318, 323, 324, 327, 331, 332, 335, 342, 350, 353, 362, 4, 17, 24, 29, 44, 53, 59, 60, 85, 92, 118, 119, 126, 135,
368, 377, 387, 389, 391 137, 141, 152, 185, 198, 228, 244, 245, 249, 250, 251,
278, 289, 294, 303, 304, 317, 328, 344, 354, 381, 382
Dermatology
116, 154, 173, 316 Neurology
30, 65, 67, 93, 98, 103, 105, 108, 112, 128, 139, 145,
Endocrinology and diabetes 190, 192, 200, 239–241, 247, 253, 255, 256, 268, 274,
5, 9, 23, 39, 46, 76, 82, 89, 92, 101, 106, 107, 127, 288, 290, 292, 307, 314, 330, 345, 365, 390, 395, 399
134, 146, 159, 164, 168, 173, 181, 199, 218, 220, 238,
242, 254, 260, 261, 273, 281, 328, 334, 372, 373, 379, Obstetric medicine
397, 401 130–132, 190, 193, 348

Environmental medicine Oncology
140 117, 216, 258, 358, 359

Gastroenterology Ophthalmology
3, 6, 19, 24, 33, 64, 72, 75, 104, 127, 133, 143, 148, 282, 345
162, 169, 182, 188, 201, 231, 276, 293, 306, 338, 339,
347, 367, 369, 371, 383, 393, 394, 400 Radiology
2, 18, 64, 88, 97, 99, 124, 183, 187, 222, 227, 252,
Genetics 280, 300, 302, 311, 343, 349, 355, 357, 360, 363
47, 85, 151, 170, 194, 195, 269, 315, 361
Respiratory medicine
Haematology 8, 14, 21, 35, 36, 37, 43, 45, 55, 56, 58, 61, 72, 79, 91,
12, 38, 49, 69, 70, 73, 86, 87, 102, 114, 115, 117, 120, 99, 111, 113, 157, 164, 196, 217, 225, 272, 279, 298,
122, 142, 156, 163, 175, 191, 204, 211, 216, 219, 233, 304, 327, 341, 349, 356, 370, 380, 384, 396
258, 263, 265, 295, 297, 299, 308, 313, 336, 346, 351,
352, 358, 376, 385, 392, 394 Rheumatology
4, 15, 17, 31, 42, 71, 77, 87, 96, 109, 141, 171, 174,
Immunology 177, 196, 198, 200, 210, 236, 264, 320, 324, 340, 364,
15, 155, 374 375, 401, 402

Infectious diseases Therapeutics/toxicology
16, 18, 26, 41, 51, 83, 88, 93, 110, 128, 142, 143, 149, 7, 8, 20, 27, 28, 36, 48, 57, 68, 77, 116, 118, 119, 165,
152, 154, 158, 166, 176, 212, 221, 225, 234, 262, 267, 172, 175, 186, 205, 206, 209, 213, 224, 251, 284, 286,
277, 280, 319, 322, 325, 337, 345, 351, 383, 386, 388 312, 316, 317, 332, 339, 366, 378

5

Abbreviations

5-HIAA 5'-hydroxyindole acetic CML chronic myeloid leukaemia FVC forced vital capacity
acid CMV cytomegalovirus GBM glomerular basement
COPD chronic obstructive
AIIRB angiotensin II receptor membrane
blocker pulmonary disease GCT giant cell tumour
CPAP continuous positive airway GFR glomerular filtration rate
AAFB acid–alcohol fast bacilli GH growth hormone
ACE angiotensin-converting pressure GHRH growth hormone releasing
CREST calcinosis, Raynaud’s
enzyme hormone
ACTH adrenocorticotrophic syndrome, oesophageal GI gastrointestinal
problems, scleroderma, GP general practitioner
hormone telangiectasia GPI glucophosphatidylinositol
ADH antidiuretic hormone CRF chronic renal failure GT glutamyltransferase
AF atrial fibrillation CRP C-reactive protein GTN glyceryl trinitrate
AIDS acquired immune- CSF cerebrospinal fluid Hb haemoglobin
CSS Churg–Strauss syndrome HbSS sickle cell anaemia
deficiency syndrome CT computed tomography HC Hereditary Copro
AIN acute interstitial nephritis CVA cerebrovascular accident
AIP acute intermittent CVP central venous pressure porphyria
CXR chest X-ray HCC hydroxy-cholecalciferol
porphyria DBP diastolic blood pressure HCM hypertrophic
ALA aminolaevulinic acid DC direct current
ALT alanine transaminase DHCC dihydroxy-cholecalciferol cardiomyopathy
DIC disseminated intravascular HCV hepatitis C virus
(SGPT) coagulation HCG human chorionic
AML acute myeloid leukaemia DIDMOAD diabetes insipidus,
AMP adenosine 5'- diabetes mellitus, optic gonadotrophin
atrophy and deafness HELLP haemolysis, elevated liver
monophosphate DM diabetes mellitus
ANA antinuclear antibody DT delerium tremens enzymes and low platelets
ANCA antineutrophil cytoplasmic DVT deep-vein thrombosis HHT hereditary haemorrhagic
EAA extrinsic allergic alveolitis
antibodies EBV Epstein–Barr virus telangiectasia
ANF antinuclear factor ECG electrocardiogram HIT heparin-induced
APCKD adult polycystic kidney EEG electroencephalogram
ELISA enzyme-linked thrombocytopenia
disease immunosorbent assay HIV human immunodeficiency
APTT activated partial EMF endomyocardial fibrosis
EMG electromyogram virus
thromboplastin time ENT ear, nose and throat HONK hypersimilar non-ketotic
AR aortic regurgitation EPO erythropoietin
ARDS adult respiratory distress ERCP endoscopic retrograde diabetic coma
cholangiopancreatogram HR heart rate
syndrome ESR erythrocyte sedimentation HRT hormone replacement
ARVC arrhythmogenic right rate
FBC full blood count therapy
ventricular cardiomyopathy FDP fibrinogen degradation HS hereditary spherocytosis
AS aortic stenosis product HSMN hereditary sensorimotor
ASD atrial septal defect FES fat embolism syndrome
ASO antistreptolysin FEV1 fixed expiration volume in neuropathy
AST aspartate transaminase 1 second HUS haemolytic uraemic
FFP fresh-frozen plasma
(SGOT) FNA fine-needle aspiration syndrome
ATN acute tubular necrosis FSH follicle stimulating ICD implantable cardioverter
AZT zidovudine hormone
BCG bacille Calmette–Guérin FTA fluorescent treponemal defibrillator
BIH benign intracranial antibody ICP intracranial pressure
INR International Normalized
hypertension
BP blood pressure Ratio
BT bleeding time IPF idiopathic pulmonary
BTS British Thoracic Society
CAH chronic active hepatitis fibrosis
CAP community acquired IVP intravenous pyelogram
IVU intravenous urogram
pneumonia JVP jugular venous pressure
CCF congestive cardiac failure KCO corrected carbon monoxide
CFTR cystic fibrosis
transfer factor
transmembrane regulator LBBB left bundle branch block
(protein) LDH lactate dehydrogenase

6

LFT liver function tests NSTEMI non-ST elevation TCAD tricyclic antidepressant
LH luteinizing hormone myocardial infarction overdose
LHON Leber’s hereditary optic TIA transient ischaemic attack
NYHA New York Heart TIBC total iron-binding capacity
neuropathy Association TIPSS transjugular intrahepatic
LHRH luteinizing hormone portosystemic shunt
OSA obstructive sleep apnoea TLC total lung capacity
releasing hormone PAN polyarteritis nodosa TLCO total lung carbon
LMWH low-molecular weight PAS periodic acid-Schiff monoxide transfer factor
PBC primary biliary cirrhosis TOE transoesophageal
heparin PBG porphobilinogen echocardiography
LQTS long QT-syndrome PCOS polycystic ovary syndrome TPA tissue plasminogen
LVEDP left ventricular end-diastolic PCR polymerase chain reaction activator
PCT porphyria cutanea tarda TPHA treponema pallidum
pressure PCV packed cell volume haemagglutination test
LVH left ventricular hypertrophy PCWP pulmonary capillary wedge TRH thyrotrophin releasing
MAHA microangiopathic hormone
pressure TSAT transferrin saturation
haemolytic anaemia PE pulmonary embolism TSH thyroid stimulating
MAOI monoamine oxidase PEFR peak expiratory flow rate hormone
PFO patent foramen ovale TT thrombin time
inhibitor PKD polycystic kidney disease TTP thrombotic
MCH mean cell haemoglobin PMLE progressive multifocal thrombocytopenic purpura
MCHC mean cell haemoglobin U&E urea and electrolytes
leucoencephalopathy URTI upper respiratory tract
content PMR polymyalgia rheumatica infection
MCV mean cell volume PNH paroxysmal nocturnal US ultrasound
MELAS mitochondrial UTI urinary tract infection
haemoglobinuria VDRL Venereal Diseases Research
encephalopathy, lactic PRL prolactin Laboratory test
acidosis, stroke-like PRV polycythaemia rubra vera VF ventricular fibrillation
syndrome PSC primary sclerosing VIP vasointestinal polypeptide
MEN multiple endocrine VMA vanilyl mandelic acid
neoplasia cholangitis VP variegate porphyria
MERRF myoclonic epilepsy and red PT prothrombin time VR ventricular rate
ragged fibres PTH parathormone or VSD ventricular septal defect
MGUS monoclonal gammopathy VT ventricular tachycardia
of undetermined parathyroid hormone WCC white cell count
significance PVE prosthetic valve WPW Wolff–Parkinson–White
MPO myeloperoxidase (syndrome)
MR mitral regurgitation endocarditis
MRA magnetic resonance RA rheumatoid arthritis
angiography RBBB right bundle branch block
MRCP magnetic resonance REM rapid eye movement
cholangiopancreatogram RMAT rapid macroagglutination
MRI magnetic resonance
imaging test
MRSA methicillin resistant RTA renal tubular acidosis
Staphylococcus aureus RV residual volume
MRV magnetic resonance SADS sudden adult death
venography
MSH melanocyte stimulating syndrome
hormone SAM systolic anterior motion of
NADPH nicotinamide adenine
dinucleotide phosphate the mitral valve
(reduced) SAP serum amyloid protein
NAPQI N-acetyl-p- SIADH syndrome of inappropriate
benzoquinoneimine
NARP neuropathy, ataxia, retinitis antidiuretic hormone
pigmentosa SLE systemic lupus
NASH non-alcoholic
steatohepatitis erythematosus
NIPPV non-invasive positive SMA smooth muscle antibody
pressure ventilation SPECT single photon emission
NSAID non-steroidal anti-
inflammatory drug computed tomography
SROS Steele–Richardson–

Olszewski syndrome
STEMI ST elevation myocardial

infarction
SVT supraventricular tachycardia
TB tuberculosis

Clinical Cases 7

Question 1 What is the best treatment to improve coronary perfusion?
a. IV Streptokinase.
A 49-year-old male presented to the Accident and b. IV Tenectoplase.
Emergency Department with a one-hour history of severe c. IV Alteplase.
central chest pain. He smoked 30 cigarettes per day. d. Half-dose tenectoplase and half-dose abciximab.
Physical examination was normal. The 12-lead ECG e. Primary coronary angioplasty.
revealed ST segment elevation in leads V1–V4. There
were no contraindications to thrombolysis.

Question 2

A 68-year-old woman presented with pain and tingling in What was the abnormality on the chest X-ray?
the left arm when she raised her hands for prolonged a. Left-sided bronchogenic carcinoma.
periods. On examination both pulses were palpable in the b. Left cervical rib.
upper limbs. The chest X-ray was abnormal. Aortography c. Retrosternal thyroid.
was performed with the arms down (2a) and with the d. Notching of the ribs.
arms up (2b). e. Widened mediastinum.

2a 2b

Question 3 Hb 9 g/dl
WCC 4.6 ϫ 109/l
A 28-year-old male presented with a six-month history of Platelets 200 ϫ 109/l
weight loss of 8 kg, generalized abdominal discomfort MCV 76 fl
and diarrhoea. On examination he was pale and slim, but ESR 38 mm/h
there were no other significant abnormalities. Sodium 141 mmol/l
Potassium 4 mmol/l
Investigations are shown. Urea 3 mmol/l
Creatinine 68 ␮mol/l
What is the diagnosis? Corrected calcium 2.02 mmol/l
a. Crohn’s disease. phosphate 0.8 mmol/l
b. Intestinal lymphangiectasia. Alkaline phosphatase 190 iu/l
c. Coeliac disease. Albumin 38 g/l
d. Small bowel lymphoma. IgA <0.1 g/l (NR 0.8–4.0 g/l)
e. Hypogammaglobulinaemia. IgG 9.0 g/l (NR 7.0–18.0 g/l)
IgM 0.6 g/l (NR 0.4–2.5 g/l)
IgA anti-endomyosial Absent
antibody

8

Answer 1

e. Primary coronary angioplasty. There have been trials evaluating the role of combined
half-dose thrombolytic therapy and half-dose platelet
Coronary reperfusion may be achieved with thrombolytic glycoprotein IIb/IIIa receptor blockers, e.g. tenectoplase
agents (which promote fibrinolysis) or by coronary plus abciximab (ASSENT 3) and reteplase plus abciximab
angioplasty. In the UK patients with ST elevation (GUSTO IV). These trials suggest that the combination
myocardial infarction are conventionally treated with may be associated with slightly higher coronary patency
thrombolytic agents. Early treatment is crucial to salvage rates and fewer ischaemic events but they have not
myocardium and reduce the risk of sudden death and demonstrated a mortality benefit. These trials have also
severe left ventricular dysfunction. Current goals for the demonstrated higher rates of intracranial bleeding in the
speed of treating with a thrombolytic agent include a elderly, hence combination therapy is not recommended
door-to-needle time of 20 minutes or a call-to-needle at present.
time of 60 minutes.
Although thrombolytic treatment is associated with a
Thrombolytic agents used commonly include significant reduction in mortality from myocardial
streptokinase, alteplase, tenectoplase and reteplase. infarction, it does have important limitations. Firstly,
Streptokinase is less favoured compared with the other greatest benefit from thrombolysis is achieved in patients
thrombolytic agents because it is less effective at restoring treated within 4 hours of the onset of symptoms. Even with
coronary perfusion and is associated with slightly worse thrombolysis normalization of blood flow is seen in only
outcomes. The GUSTO I study compared front-loaded 50–60% of cases. Recurrent ischaemia occurs in 30% of
alteplase therapy with streptokinase in patients with ST cases and frank thrombotic coronary occlusion in 5–15%.
EMI. Alteplase was superior to streptokinase in reducing Re-infarction occurs in up to 5% of cases while in hospital.
mortality (1% absolute reduction in mortality at 30 days Also major bleeding is recognized in 2–3% of cases. For
with alteplase) and was associated with greater coronary these reasons several trials were set up comparing primary
patency rates. In the GUSTO trial the benefit was angioplasty with thrombolysis in STEMI.
greatest in patients aged under 75 years and those with
anterior myocardial infarction. However, streptokinase is Primary angioplasty is superior to thrombolysis. It is
still used extensively in developing countries and in many associated with lower mortality and lower re-infarction
hospitals in the UK. Alteplase, tenectoplase and reteplase rates. The likelihood of a pre-discharge positive exercise
appear to be equally effective. Tenectoplase and reteplase test is also reduced by primary angioplasty. In hospitals
are easier to administer (as a single bolus). where facilities for primary angioplasty are available,
primary angioplasty should be considered over
thrombolysis. Best results occur when the door-to-
balloon time is less than 2 hours.

Answer 2

b. Left cervical rib. ribs are common in the normal population and are
usually asymptomatic. In rare circumstances a cervical rib
There is mechanical occlusion of the left subclavian artery may cause pressure on the subclavian vessels and the
on raising the left arm due to a left cervical rib. Cervical brachial plexus causing transient vascular insufficiency or
paraesthesiae in the upper limb.

Answer 3

c. Coeliac disease. endomyosial antibodies are IgA antibodies, therefore
they will not be detected in patients with low IgA
Diarrhoea, weight loss, abdominal discomfort and antibody levels. Since coeliac disease is also associated
isolated IgA deficiency are highly suggestive of coeliac with IgA deficiency it is important to be aware of serum
disease. Anti-endomyosial antibodies are highly sensitive IgA levels before interpreting anti-endomyosial
and specific for the diagnosis of coeliac disease. Anti- antibodies in patients with malabsorption. (See Question
276.)

Clinical Cases 9

Question 4 Hb 7 g/dl

A 53-year-old male was admitted to hospital with a two- WCC 11 ϫ 109/1
week history of coughing and breathlessness. Apart from
a longstanding history of mild asthma he had been (neutrophils 8 ϫ 109/l,
relatively well with respect to the respiratory tract. He
had been on a skiing trip six weeks previously, without lymphocytes 2 ϫ 109/l,
any respiratory problems.
eosinophils 1 ϫ 109/l)
He had a past history of depression, for which he took
lithium five years ago, and suffered from occasional ESR 38 mm/h
tension headaches, for which he took simple analgesia.
Sodium 134 mmol/l
On examination he appeared pale and unwell. His
heart rate was 90 beats/min and regular. His blood Potassium 4.6 mmol/l
pressure measured 160/94 mmHg. The JVP was not
raised. Both heart sounds were normal and the chest was Urea 48 mmol/l
clear. Abdominal examination did not reveal any
abnormality. Urinalysis demonstrated blood ++ and Creatinine 798 mmol /l
protein ++.
Renal ultrasound
Investigations performed in hospital are shown.
Both kidneys measured 12 cm: there was no
What is the most likely diagnosis?
a. Rapidly progressive glomerulonephritis. evidence of ureteric obstruction.
b. Analgesic nephropathy.
c. Renal amyloidosis.
d. Churg–Strauss syndrome.
e. IgA nephritis.

Question 5 FBC Normal

A 52-year-old male presented with impotence. He had a Sodium 135 mmol/l
four-year history of insulin-dependent diabetes mellitus.
There was no history of headaches or vomiting. The Potassium 4 mmol/l
patient was a non-smoker and did not consume alcohol.
Apart from insulin he took simple analgesia for joint Urea 6 mmol/l
pains.
Creatinine 100 mmol/l
Investigations are shown.
Bilirubin 12 mmol/l
What test would you perform to confirm the
diagnosis? AST 200 iu/l

a. MRI scan of the brain. ALT 220 iu/l
b. Serum prolactin level.
c. Serum ferritin. Alkaline phosphatase 128 iu/l
d. Dynamic pituitary function tests.
e. Liver ultrasound. Albumin 8 g/l

Thyroxine 100 nmol/l

TSH 2.6 mu/l

Testosterone 7 nmol/l (NR 10–35 nmol/l)

LH 1.5 iu/l (NR 1–10 iu/l)
FSH
LHRH test: 1 iu/l NR 1–7 iu/l)
LH
FSH 20 min: 60 min:

3 iu/l 2 iu/l

2 iu/l 2 iu/l

10

Answer 4

d. Churg–Strauss syndrome. Table A Phases of Churg–Strauss syndrome:

The patient has a past history of asthma, eosinophilia and 1. The prodromal phase, which may be present for
rapidly progressive glomerulonephritis. The most probable years and comprises of rhinitis, nasal polyposis and
diagnosis is Churg–Strauss syndrome. The assumption that frequently asthma.
he probably has rapidly progressive glomerulonephritis is
based on the fact that he was well enough to ski six weeks 2. The eosinophilic phase, which can remit and recur
ago, which would be highly unlikely in a patient with end- for years. It is characterized by the onset of
stage renal disease. The identification of normal-sized peripheral blood and tissue eosinophilia, resembling
kidneys during renal ultrasonography supports acute rather Loeffler’s syndrome, chronic eosinophilic
than chronic renal failure (Table A). pneumonia or eosinophilic gastroenteritis.

Churg–Strauss syndrome is a small-vessel multi-system 3. The vasculitic phase, which usually occurs in the
vasculitis characterized by cutaneous vasculitic lesions, third or fourth decades of life and is characterized
eosinophilia (usually <2.0 ϫ 109/l), asthma (usually by a life-threatening systemic vasculitis of small and
mild), mononeuritis or polyneuropathy and rarely occasionally medium-sized vessels. This phase is
glomerulonephritis (10% of cases). Gastrointestinal and associated with constitutional symptoms and signs,
cardiac involvement is recognized. fever and weight loss.

Pulmonary findings dominate the clinical presentation Table B American College of Rheumatology 1990
with paroxysmal asthma attacks and presence of fleeting criteria for Churg–Strauss syndrome
pulmonary infiltrates. Asthma is the cardinal feature and may
be present for years before overt features of a multi-system The presence of four or more of the manifestations
vasculitis become apparent. Skin lesions, which include below is highly indicative of Churg–Strauss syndrome:
purpura and cutaneous and subcutaneous nodules, occur in • Asthma
up to 70% of patients. Gastrointestinal complications include • Eosinophilia (10% on WCC differential)
mesenteric ischaemia or gastrointestinal haemorrhage. • Mononeuropathy or polyneuropathy
Cardiac involvement is characterized by myo-pericarditis. • Migratory or transient pulmonary infiltrates
• Systemic vasculitis (cardiac, renal, hepatic)
The diagnosis is usually clinical and supported by the • Extravascular eosinophils on a biopsy including
presence of a necrotizing granulomatous vasculitis with
extravascular eosinophilic infiltration on lung, renal or sural artery, arteriole or venule
biopsy. The American College or Rheumatology criteria
for the diagnosis of Churg–Strauss syndrome are tabulated Table C Causes of renal failure and eosinophilia
(Table B). Serum ANCA (MPO subset) are elevated but
this finding is also present in microscopic polyangitis. • Rapidly progressive glomerulonephritis
• Churg–Strauss syndrome
The prognosis of untreated CSS is poor, with a • Acute tubulo-interstitial nephritis
reported five-year survival rate of only 25%. Corticosteroid • Cholesterol micro-emboli
therapy has been reported to increase the five-year
survival rate to more than 50%. In patients with acute associated with eosinophilia. Causes of renal failure and
vasculitis the combination of cyclophosphamide and eosinophilia are tabulated (Table C).
prednisone is superior to prednisolone alone.
The history of analgesia for headaches raises the
Although rapidly progressive glomerulonephritis also possibility of analgesic nephropathy as the cause of his
features in the answer options section, the presence of presentation; however, analgesic nephropathy is usually
asthma and eosinophilia make Churg–Strauss syndrome insidious and many patients present for the first time with
the best answer. It is worth noting however, that rapidly renal failure. The majority have abnormalities on renal
progressive glomerulonephritis may also rarely be ultrasound scans. Analgesic nephropathy alone does not
explain asthma or eosinophilia.

Answer 5

c. Serum ferritin. involvement, and abnormal liver function is consistent
with hepatic infiltration.
The clinical features and the data are consistent with the
diagnosis of idiopathic haemochromatosis. The insulin- The patient has a low testosterone level with an
dependent diabetes mellitus suggests pancreatic inappropriately low gonadotrophin response indicating
secondary hypogonadism due to excessive iron deposition
in the pituitary. Secondary hypogonadism is the most

Clinical Cases 11

common endocrine deficiency in hereditary haemo- ferritin concentrations even in the absence of iron overload.
chromatosis. Primary hypogonadism due to testicular Hepatic iron overload in haemochromatosis is associated
iron deposition may occur with this disorder but is much
less common than secondary hypogonadism. with an increased risk of hepatocellular carcinoma. Patients
with haemochromatosis are also at increased risk of
In the context of the question, a serum ferritin level hypothyroidism and are susceptible to certain infections
>500 mg/l would be diagnostic of primary haemo- from siderophoric (iron-loving) organisms such as Listeria
chromatosis. Alcohol-related liver disease, chronic viral spp., Yersinia enterocolitica and Vibrio vulnificus, which are
hepatitis, non-alcoholic steatohepatitis and porphyria picked up from eating uncooked seafood.
cutanea tarda also cause liver disease and increased serum

Question 6 Hb 12.6 g/dl

A 38-year-old English male was investigated after he was WCC 8 ϫ 109/l
found to have an abnormal liver function test during a
health insurance medical check. He worked in an Platelets 210 ϫ 109/l
information technology firm. Apart from occasional
fatigue he was well. He consumed less than 20 units of MCV 90 fl
alcohol per week. The patient had only travelled out of
Europe twice and on both occasions he had been to Sodium 136 mmol/l
North America. He took very infrequent paracetamol for
aches and pains in his ankles and knees. There was no Potassium 4.1 mmol/l
history of hepatitis or transfusion or blood products. He
had been married for 5 years. Systemic enquiry revealed Urea 6 mmol/l
infrequent episodes of loose stool for almost 4 years.
Creatinine 100 mmol/l
On examination he appeared well. There were no
stigmata of chronic liver disease. Abdominal examination AST 60 iu/l (NR 10–40 iu/l)
revealed a palpable liver edge 3 cm below the costal
margin. There were no other masses. Examination of the ALT 78 iu/l (NR 5–30 iu/l)
central nervous system was normal.
Alkaline phosphatase 350 iu/l (NR 25–100 iu/l)
Investigations were as shown.
Bilirubin 22 mmol/l (NR 2–17 μmol/l)
What is the most probable diagnosis?
a. Autoimmune hepatitis. Albumin 38 g/l (NR 34–48 g/l)
b. Primary sclerosing cholangitis.
c. Primary biliary cirrhosis. Total cholesterol 5.2 mmol/l
d. Haemochromatosis.
e. Wilson’s disease. Triglyceride 3.1 mmol/l

Blood glucose 6 mmol/l

Ferritin 256 mg/l (NR 15–250 mg/l)

Serum Fe 28 mmol/l

(NR 14–32 mmol/l)

TIBC 50 mmol/l

(NR 40–80 mmol/l)

Serum Slightly reduced

caeruloplasmin

24-hr urine copper Slightly elevated

IgG 19 g/l (NR 7–18 g/l)

IgA 4.2 g/l (NR 0.8–4.0 g/l)

IgM 5.0 g/l (NR 0.4–2.5 g/l)

Anti-nuclear Positive 1/32

antibodies

Smooth muscle Not detected

antibodies

Antimitochondrial Not detected

antibodies

Hep B sAg Not detected

Hep C virus Not detected

antibodies

Abdominal ultrasound Normal

Question 7 What is the best marker of prognosis?
a. Serum aspartase transaminase.
A 17-year-old girl presented with jaundice three days b. Serum alkaline phosphatase.
after having taken a paracetamol and alcohol overdose c. Serum bilirubin.
during an argument with her boyfriend. d. Prothrombin time.
e. Paracetamol level.

12

Answer 6

b. Primary sclerosing cholangitis. progression of the disorder. Patients are treated with
cholestyramine to reduce pruritus. Fat-soluble vitamin
This is a relatively difficult question. The history of loose supplementation is necessary owing to steatorrhoea.
stool is crucial in making the diagnosis in this particular Antibiotic prophylaxis during instrumentation of the
case in the absence of data from the ERCP. Diarrhoea biliary tree is mandatory to reduce the risk of bacterial
and biochemical evidence of cholestasis (alkaline cholangitis. Ciprofloxacin is the prophylactic antibiotic
phosphatase greater than transaminases) should lead to drug of choice prior to ERCP. Biliary stenting may
the clinical suspicion of primary sclerosing cholangitis improve biochemistry and symptoms; however, the
(PSC). The aetiology of PSC is unknown but definitive treatment for PSC is hepatic transplantation.
immunological destruction of intra- and extra-hepatic
bile ducts is the main pathological feature. 90% of PSC is Although a cholestatic picture is also recognized in
associated with inflammatory bowel disease, particularly primary biliary cirrhosis, alcohol abuse and viral hepatitis
ulcerative colitis, and hence the importance of the there is nothing in the history or investigations to indicate
intermittent diarrhoea. Ulcerative colitis is the most these conditions as the cause of his illness. Primary biliary
frequent association with primary sclerosing cholangitis. cirrhosis affects mainly females in the fifth decade
A raised alkaline phosphatase level in a patient with onwards. Furthermore, the absence of anti-mitochondrial
ulcerative colitis (in the absence of bone disease) should antibodies is against the diagnosis. The ferritin is modestly
raise the possibility of PSC. The frequency of PSC is raised but not high enough to suggest hereditary
inversely proportional to the severity of ulcerative colitis. haemochromatosis. High ferritin levels are also a feature
Other associations of PSC include coeliac disease. of chronic viral hepatitis, alcohol-related hepatitis and
non-alcoholic steato-hepatitis. Hypergammaglobulinaemia
Patients with PSC may be asymptomatic at pre- and raised autoantibody titres are features of primary
sentation but can present with advanced liver disease. sclerosing cholangitis but also occur in other
Fatigue and pruritus are common complaints as with the immunological liver disorders such as chronic active viral
other cholestatic disorders. Approximately one-fifth of hepatitis, auto-immune hepatitis and biliary cirrhosis.
the patients also complain of right upper quadrant pain.
Patients with cholestasis also have lowish caerulo-
The diagnosis is confirmed with ERCP that shows plasmin levels and increased blood and urine copper
strictures within biliary ducts. Complications are those of levels. The abnormal copper metabolism in this case
chronic cholestasis, notably statorrhoea, fat-soluble should not lead to the candidate diagnosing Wilson’s
vitamin malabsorption, large biliary strictures, cholangitis, disease, since there are many features above to indicate
cholangiocarcinoma and colonic carcinoma. There are no PSC. Furthermore, patients with Wilson’s disease usually
effective pharmacological agents that greatly retard the have a hepatitic biochemistry picture and often have co-
existing neuro-psychiatric disease.

Answer 7 Important risk markers for severe hepatic injury after
paracetamol overdose include a PT >20 seconds 24 h
d. Prothrombin time. after ingestion, pH <7.3 and creatinine >300 ␮mol/l.
(See Questions 27 and 206.)

Clinical Cases 13

Question 8

A 16-year-old girl presented with an 18-month history of crackles in the mid and lower zones. Repeat lung
progressive breathlessness on exertion. On admission she function tests revealed an FEV1/FVC ratio of 86% and a
was breathless at rest. She had a past history of acute transfer factor of 60% predicted.
myeloid leukaemia, for which she had been treated with
six courses of chemotherapy, followed by bone marrow What is the cause of her symptoms?
transplantation supplemented with radiotherapy and a. Previous radiotherapy.
cyclophosphamide treatment five years ago. She was b. CMV pneumonitis.
regularly followed up in the haematology clinic. Lung c. Pneumocystis carinii pneumonia.
function tests three years ago revealed an FEV1/FVC d. Cyclophosphamide-induced lung fibrosis.
ratio of 80%. On examination she was breathless at rest, e. Severe anaemia.
and cyanosed. There was no evidence of clubbing.
Auscultation of the lung fields revealed fine inspiratory

Question 9

A 21-year-old man was admitted to the intensive care a severe head injury. He required ventilation.

unit after a road traffic accident during which he suffered Investigations are shown.

Sodium 128 mmol/l What is the cause of the hyponatraemia?
Potassium 3.6 mmol/l a. Hypopituitarism.
Creatinine 81 mmol/l b. Addison’s disease.
Urea 4 mmol/l c. Syndrome of inappropriate ADH secretion.
Thyroxine 30 nmol/l d. Hypothyroidism.
TSH 2 mu/l e. Cushing’s syndrome.
Serum cortisol 1000 nmol/l
(NR 170–700 nmol/l)

Question 10 How would you alter her treatment?
a. Add spironolactone.
A 40-year-old woman with dilated cardiomyopathy is b. Substitute ramipril with losartan.
seen in the heart failure clinic complaining of a persistent c. Reduce carvedilol to 3.125 mg twice daily.
dry cough. Her exercise capacity is 1 mile while walking d. Double the dose of furosemide.
on the flat. She can climb two flights of stairs without e. Add digoxin.
difficulty. Her medication consists of ramipril 10 mg
daily, aspirin 75 mg daily, carvedilol 6.25 mg twice daily
and frusemide 40 mg daily. On examination her heart
rate is 70 beats/min and her blood pressure is
100/60 mmHg. Both heart sounds are normal and the
chest is clear.

14

Answer 8

d. Cyclophosphamide-induced lung fibrosis. Cyclophosphamide therapy can also result in an acute
pneumonitis during treatment with the drug that causes
The patient presents with progressive symptoms cough, dyspnoea, hypoxia and bilateral nodular opacities
associated with a restrictive lung defect and a low in the upper zones of the lung. Acute cyclophosphamide-
transfer factor. The findings are most consistent with induced pneumonitis responds to cessation of the drug
cyclophosphamide-induced pulmonary fibrosis. and corticosteroid therapy.

Cyclophosphamide-induced lung fibrosis is rare and is The differential diagnosis in this case is radiation-
most likely to occur in patients who have had concomitant induced fibrosis. Radiotherapy to the pulmonary area
pulmonary radiation therapy or have taken other drugs usually causes a pneumonitis that presents with cough,
associated with pulmonary toxicity. The disorder usually dyspnoea, a restrictive lung defect and low transfer factor.
occurs in patients who have been taking low doses for It is more common in patients also taking cyclo-
relatively prolonged periods (over six months) and phosphamide or bleomycin. Unlike cyclophosphamide-
presents several years after cessation of the drug and induced pulmonary fibrosis the condition is not
hence the deterioration of symptoms with time. The associated with an inexorable decline. Indeed many
disorder has a relentless progression and inevitably results patients show improvement in symptoms and objective
in terminal respiratory failure. It is minimally responsive pulmonary function testing within 18 months of
to corticosteroids. Fine end-inspiratory crackles and stopping radiotherapy.
clubbing do not usually form part of the clinical
spectrum. Causes of drug-induced pulmonary fibrosis

The diagnosis is clinical. Chest X-ray reveals reticulo- • Cyclophosphamide
nodular shadowing of the upper zones. Lung function • Busulphan
tests demonstrate a restrictive lung defect. Lung biopsy is • Methysergide
not helpful. • Methotrexate
• Amiodarone
Cyclophosphamide per se is not toxic to the lungs; • Nitrofurantoin
however, it is metabolized in the liver to toxic • Minocycline
metabolites such as hydroxycyclophosphamide, acrolein • Ethambutol
and phosphoramide mustard, which are responsible for • Penicillamine
pulmonary damage. Genetic factors may play a role in
determining which individuals develop pulmonary
fibrosis after exposure to the drug.

Answer 9

c. Syndrome of inappropriate ADH secretion. be euthyroid, therefore the term sick euthyroid syndrome
was used to describe these biochemical abnormalities.
The patient has a low sodium concentration in the There is evidence now that these abnormalities represent
context of a head injury. The thyroid function tests genuine acquired transient central hypothyroidism.
suggest the possibility of a secondary hypothyroidism, i.e. Treatment with thyroxine in these situations is not
a low TSH and a low thyroxine concentration, and hence helpful and may be harmful. It is thought that these
the possibility of damage to the pituitary. However, the changes in thyroid function during severe illness may be
very high cortisol level indicates that pituitary function is protective by preventing excessive tissue catabolism.
probably normal (high ACTH production secondary to Thyroid function tests should be repeated after at least six
stress) and therefore the abnormal thyroid function tests weeks following recovery.
represent sick euthyroid syndrome. Low T4, T3 and TSH
levels are recognized in critically ill patients with non- Critical illness may also reduce T4 by reducing thyroid
thyroid illnesses. Originally such patients were thought to binding globulin levels, and T3 is rapidly reduced owing
to inhibition of peripheral de-iodination of T4.

Clinical Cases 15

Answer 10

b. Substitute ramipril with losartan. The study revealed similar mortality rates and similar
rates of progression of heart failure when comparing
The patient is in NYHA functional class II with respect to patients on losartan 50 mg daily with those prescribed
her symptoms. She is on the correct dose of ramipril and captopril 50 mg three times daily. The study suggests
is appropriately being treated with a beta-blocker. The that losartan is as effective as ACE inhibitors in the
dry cough that the patient is experiencing is almost management of heart failure. However, the use of
certainly the side-effect of ramipril. Angiotensin- losartan in heart failure is still currently reserved for
converting enzyme inhibitors are associated with a dry patients who develop side-effects to ACE inhibitors. A
cough in 15–20% of patients owing to increases in recent study evaluating the role of angiotensin receptor
circulating bradykinin levels. In such patients the ACE blockers (CHARM study; evaluated candesartan) in
inhibitor should be stopped and substituted with an patients with heart failure showed reduced hospitalization
angiotensin receptor blocker such as losartan. The rates and mortality in heart failure patients who were on
efficacy of losartan compared with an ACE inhibitor candesartan instead of an ACE inhibitor, or candesartan
(captopril) was fully evaluated in the ELITE II study. as additional therapy to an ACE inhibitor.

Question 11

A 60-year-old male was admitted to the coronary care unit after the procedure, and an exercise stress test performed
with central chest pain. Physical examination was normal. four weeks after the procedure was negative for
The blood pressure measured 110/68 mmHg. The 12- myocardial ischaemia for 10 minutes.
lead ECG was normal and the troponin T level was not
raised. The blood sugar was normal. The cholesterol level What other medication should the patient receive to
on admission was 6.3 mmol/l. The patient underwent an improve his cardiovascular prognosis?
exercise stress test that was positive. A subsequent
coronary angiogram revealed an 80% stenosis in the a. Atenolol.
proximal aspect of the left anterior descending artery that b. Ramipril.
was successfully treated with a coronary artery stent. c. Candesartan.
Echocardiography revealed a normal-sized left ventricle d. No further treatment required.
with good systolic function. The patient was discharged e. Isosorbide dinitrate.
home on aspirin 75 mg daily, clopidogrel 75 mg daily and
simvastatin 40 mg daily. He had been completely pain free

Question 12

A 62-year-old obese male with a known medical history On examination he was obese. His chest was clear and
of hypertension presented with generalized headaches examination of the abdomen did not reveal any
and lethargy. He was taking bendroflumethiazide, abnormality.
2.5 mg once daily for hypertension. The only other past
medical history included a left-sided deep vein Investigations are shown.
thrombosis six months previously. There was no history
of alcohol abuse or smoking. Hb 20 g/dl
MCV
What is the cause of his symptoms? WCC 88 fl
a. Obstructive sleep apnoea. Platelets 15 ϫ 109/l
b. Gaissbock’s syndrome. PCV 500 ϫ 109/l
c. Polycythaemia rubra vera. Sodium 0.66 l/l
d. Renal cell carcinoma. Potassium
e. Chronic hypoxaemia. Urea 141 mmol/l
Creatinine
Urate 4.2 mmol/l

8 mmol/l
110 ␮mol/l
0.44 mmol/l

16

Answer 11

b. Ramipril. risk population with normal left ventricular function.
Based on this study all patients with coronary artery
The Heart Outcomes Prevention Evaluation Study disease, cerebrovascular disease, peripheral vascular
(HOPE) evaluated the role of angiotensin-converting disease and diabetes mellitus plus one other risk factor for
enzyme inhibitors (ramipril) in populations at high risk of coronary artery disease should be prescribed an ACE
cardiovascular events without any evidence of left inhibitor, specifically ramipril.
ventricular dysfunction. The study assessed 9297 high-
risk patients, defined as (1) aged >55 years; (2) history of The patient should remain on aspirin for life and take
coronary artery disease, stroke or peripheral vascular clopidogrel for a year following deployment of a stent.
disease; or (3) diabetes mellitus and at least one risk The CURE study showed that aspirin and clopidogrel
factor for coronary artery disease including hypertension, together were associated with a lower incidence of
increased total cholesterol, smoking and micro- myocardial infarction and death in patients with unstable
albuminuria. The patients were randomized to ramipril angina and non-ST elevation myocardial infarction
10 mg daily or placebo. The primary outcome was a compared with aspirin alone for up to a year.
combined endpoint of myocardial infarction, stroke or
cardiovascular death. The mean follow up was five years. The patient no longer has subjective or objective
evidence of myocardial ischaemia, and in the absence of
Patients treated with ramipril had a 14% event rate of hypertension or left ventricular dysfunction there is no
the combined morbidity and mortality endpoint whereas indication for a beta-blocker.
placebo-treated patients had a 17.8% event rate. The 21%
decrease in events was seen in all pre-specified groups, Nitrates do not alter prognosis in coronary artery
indicating that ACE inhibitor therapy with ramipril disease. There is no evidence as yet that angiotensin
significantly reduces morbidity and mortality in a high- receptor blockers improve cardiovascular prognosis in
patients with coronary artery disease in the absence of
hypertension or left ventricular dysfunction.

Answer 12

c. Polycythaemia rubra vera. Many patients with polycythaemia rubra vera have
splenomegaly; however, a palpable spleen is absent in
The high Hb is suggestive of polycythaemia. There is approximately one third of patients.
nothing in the history to indicate a secondary cause, e.g.
hypoxia, renal carcinoma, adrenal tumour. Although he Criteria for the diagnosis of polycythaemia rubra
was obese, there was nothing else in the history to allow vera
the diagnosis of obstructive sleep apnoea.
Raised red cell mass and normal pO2 with either
The high white cell count and platelet count favour splenomegaly or two of the following:
primary polycythaemia (polycythaemia rubra vera). • WCC >12 ϫ 109/l
Headache and lethargy are common symptoms of • Platelets >400 ϫ 109/l
polycythaemia rubra vera. Polycythaemia rubra vera • Raised B12 binding protein
causes lethargy due to hyperviscosity and raised • Low neutrophil alkaline phosphatase
interleukin-6 levels. Other classic features include visual
disturbance, abdominal pain and pruritus. concentration

(See Questions 39, 73 and 211.)

Clinical Cases 17

Question 13 What is the most probable underlying diagnosis?
a. Coarctation of the aorta.
The ECG below was taken from a young boy who b. Dextrocardia.
experienced syncope. On examination he had a systolic c. Pulmonary stenosis.
murmur. d. Wolff–Parkinson–White syndrome.
e. Hypertrophic cardiomyopathy.

13

Question 14 What is the management?
a. Admit and observe for 24 hours.
An 18-year-old male was admitted with sudden sharp b. Attempt aspiration of pneumothorax.
pain in the left infrascapular area. He was not breathless c. Prescribe 100% oxygen for a few hours.
on mild exertion. He was usually fit and well. He was an d. Insert chest drain.
occasional smoker. There was no history of respiratory e. Allow home and repeat CXR after a week.
problems. On examination there was reduced air entry at
the left lung base. The oxygen saturation on air was 96%.
The CXR revealed a left-sided pneumothorax. There was
less than 2 cm rim of air between the edge of the lung
and the ribs.

18

Answer 13

c. Pulmonary stenosis. ventricular hypertrophy. The answer that would fit with
all the information is pulmonary stenosis. Coarctation of
The patient has a systolic murmur. The ECG shows the aorta and hypertophic cardiomyopathy are associated
right axis deviation, a dominant R wave in V1 and with left ventricular hypertrophy. The absence of a short
relatively prominent S waves in V5 and V6. The sum of PR interval and delta waves are against the diagnosis of
the R in V1 and in V6 is > 1.25 mV which indicates right WPW syndrome.

Answer 14 Management of pneumothorax
Spontaneous
e. Allow home and repeat CXR after a week. pneumothorax

The question tests knowledge of the guidelines for the < 2cm rim of air on CXR
management of pneumothorax set by the British Minimal symptoms
Thoracic Society.
Yes No
The patient has a relatively small pneumothorax
(<2 cm rim of air between lung and ribs) with minimal Allow home Aspirate
symptoms and can walk slowly without becoming Repeat CXR in
breathless. There is no history to suggest chronic lung If unsuccessful,
disease. In such a case no treatment is recommended and 7–10 days repeat aspiration.
the patient may be discharged. Patients are advised not to If still unsuccessful,
over-exert themselves and to return if they develop Successful insert chest drain
breathlessness. A repeat CXR is recommended after a
week to ensure that the pneumothorax has resolved.

If the patient has a pneumothorax >2 cm rim of air
between the lung and the chest wall on the CXR, or has
pain or dyspnoea at rest or on minimal exertion then
aspiration is recommended. If aspiration is successful the
patient is allowed home and reviewed with repeat CXR in
one week. If aspiration is unsuccessful a second attempt is
made at aspiration. If the lung still remains deflated then
insertion of a chest drain is recommended.

In patients with chronic lung disease the following
criteria should be used to decide whether aspiration or
insertion of a chest drain is the first procedure of choice.
Patients aged <50 years, who are relatively asymptomatic
and have a small pneumothorax, should be aspirated and
observed in hospital for 24 hours (assuming aspiration is
successful). If aspiration is unsuccessful in this group of
patients then insertion of a chest drain is advised. In
patients aged >50 years, with symptoms and with larger
pneumothoraces (>2 cm air between lung and chest wall)
a chest drain is necessary.

Clinical Cases 19

Question 15 Investigations are shown.

A 44-year-old was seen in the rheumatology clinic in Hb 10 g/dl
December complaining of malaise, joint pains and WCC 9 ϫ 109/l
tingling in the hands and feet. She had been diagnosed as Platelets 490 ϫ 109/l
having Raynaud’s phenomenon several years ago. The ESR 90 mm/h
patient had consulted several doctors for intermittent Sodium 139 mmol/l
malaise and joint pains. There was no history of night Potassium 4.2 mmol/l
sweats, dyspnoea, or problems with swallowing. The Urea 9 mol/l
patient took paracetamol on a PRN basis for joint pains. Creatinine 140 ␮mol/l
Bilirubin 15 mmol/l
On examination she had palpable purpura on the AST 90 iu/l
thighs and arms. There was no obvious evidence of joint Alkaline phosphatase 122 iu/l
swelling. Abdominal examination revealed hepatomegaly Albumin 33 g/l
palpable 3 cm below the costal margin. Neurological Rheumatoid factor IgM Positive (titre 1/640)
examination revealed decreased sensation in the hands C3 0.2 g/l (NR 0.55–1.2 g/l)
and feet. The blood pressure was 110/80 mmHg. C4 0.09 g/l (NR 0.2–0.5 g/l)
Hep C Virus AB Positive
What is the best management of the patient’s illness? Hep B sAg Negative
a. Prednisolone. Urinalysis Blood +
b. Cyclophosphamide. Protein ++
c. Chlorambucil.
d. Pegylated interferon-␣ plus ribavarin.
e. Plasmapharesis.

Question 16

A 30-year-old businessman developed sudden onset of On examination his temperature was 38.6°C. There
fever, sore throat, diarrhoea and myalgia. Over the next was cervical lymphadenopathy. Inspection of the oral
three days he noticed a widespread rash affecting his face, cavity revealed several painful ulcers affecting the tongue.
trunk, palms and soles. He was usually fit and well and The pharynx was oedematous and red with minimal
had only consulted his GP once in the past 10 years for a tonsillar exudates. The chest was clear. Abdominal
typhoid vaccine before travelling to India. Over the past examination was normal.
four months he had established business links with a
company in Thailand and had visited the country on Investigations are shown.
three occasions. His last visit to Thailand was eight weeks
previously. He was married with two young children. He Hb 13 g/dl
was not taking any medications and had no history of WCC 11 ϫ 109/l
drug allergy. (neutrophils 6 ϫ 109/l,
Platelets lymphocytes 4 ϫ 109/l)
What is the diagnosis? Monspot test 130 ϫ 109/l
a. Acute HIV infection. Sodium Negative
b. Secondary syphilis. Potassium
c. Acute hepatitis infection. Urea 135 mmol/l
d. Infectious mononucleosis. Creatinine
e. Acute CMV infection. Bilirubin 3.8 mmol/l
ALT
AST 6 mmol/l
80 ␮mol/l
23 ␮mol/l
45 iu/l

49 iu/l

20

Answer 15

d. Pegylated interferon-␣ plus ribavarin. with mixed essential cryoglobulinaemia. Cryoglobulins
are immunoglobulins that precipitate in the cold. They
This is a difficult question; however, the clue lies in the are associated with auto-immune haemolysis, Raynaud’s
fact that the patient has evidence of current or previous disease (in severe cases they can cause acronecrosis),
infection with hepatitis virus and has Raynaud’s vasculitis, peripheral neuropathy, glomerulonephritis and
phenomenon, palpable purpura (vasculitis), neuropathy hepatosplenomegaly. Complement is reduced. HCV is
and hypocomplementaemia. The diagnosis is consistent thought to play an aetiological role in the development in
type II and type III cryoglobulinaemia.

Types of cryoglobulinaemia Associated condition(s)
Multiple myeloma
Type Immunoglobulins Waldenstrom’s macroglobulinaemia
I Monoclonal immunoglobulin Hepatitis C and hepatitis B

II Polyclonal IgG and monoclonal Chronic inflammation
rheumatoid factor IgM Hepatitis C
Lymphoproliferative disease
III Mixed IgG and polyclonal
rheumatoid factor

The diagnosis is based upon history, skin biopsy (if cyclophosphamide are effective. Chlorambucil has also
purpura present), hypocomplementaemia and presence of been used with success. When cryoglobulinaemia is
cryoglobulins. Investigation for cryoglobulinaemia secondary to HCV infection, the treatment of choice
should always include serology for hepatitis C infection. includes the combination of pegylated interferon-a and
Treatment for acute cryoglobulinaemia causing severe ribavarin. Ribavarin should be used with caution in
renal impairment or acronecrosis is plasmapharesis, patients with renal failure.
though in less acute situations prednisolone and

Answer 16

a. Acute HIV infection. EBV and CMV infection. Furthermore the rash in
infectious mononucleosis is usually an idiosyncratic
The main differential diagnosis is between infectious reaction to ampicillin whereas it is part of HIV
mononucleosis, CMV infection and acute HIV infection. seroconverson. The main clinical features differentiating
All three are associated with sore throat, rash, fever and infectious mononucleosis from acute HIV infection are
atypical lymphocytes. Mouth ulcers are usually absent in tabulated below. The rash in CMV infection usually spares
the palms and soles. (See Question 325.)

Differentiation between infectious mononucleosis and acute HIV infection

Parameter Infections mononucleosis HIV infection
Onset of symptoms Over a few days Abrupt
Mouth ulcers Absent usually Often present
Rash Usually secondary to ampicillin Part of HIV seroconversion
Diarrhoea Unusual Common
Tonsillar exudates Prominent Mild
White cell count May be elevated Elevated or suppressed
Atypical lymphocytes Frequent (90%) and numerous Present in 50%
Transaminitis Common Common
Thrombocytopenia Common Common

Clinical Cases 21

Question 17

A 69-year-old woman with rheumatoid arthritis heart sounds were normal and the chest was clear.
presented with swollen ankles. She was diagnosed as Abdominal examination was normal. Inspection of the
having rheumatoid arthritis over 18 years ago and had lower limbs revealed pitting oedema.
been relatively well controlled on non-steroidal anti-
inflammatory drugs until six months ago, when her joint Investigations are shown.
pains and swelling required the addition of penicillamine
to control her symptoms. The patient had a past history Hb 11 g/dl
of hypertension, for which she took bendroflumethiazide. WCC 5 ϫ 109/l
Platelets 190 ϫ109/l
On examination she had symmetrical joint deformities Sodium 134 mmol/l
consistent with rheumatoid arthritis. The heart rate was Potassium
90 beats/min and irregular. Her blood pressure Urea 4.5 mmol/l
measured 140/90 mmHg. The JVP was not raised. Both Creatinine
Bilirubin 6 mmol/l
What is the management? Alkaline phosphatase 70 ␮mol/l
a. Stop penicillamine. Albumin 11␮mol/l
b. Start prednisolone. Urinalysis 100 iu/l
c. Start ACE inhibitor therapy.
d. Arrange renal biopsy. 26 g/l
e. Arrange IVU. Protein ϩϩϩ

Question 18 18

A 59-year-old female presented with weakness of both
legs. An MRI scan of the spine is shown (18).

What is the cause of her symptoms?
a. Syringomyelia.
b. Paravertebral abscess.
c. Thoracic disc prolapse.
d. Metastatic spinal cord compression.
e. Extradural meningioma.

22

Answer 17

a. Stop penicillamine. initiation of penicillamine points to a drug-induced
membranous nephropathy.
The patient has heavy proteinuria and gives a relatively
recent history of onset of swollen ankles shortly after Other causes of renal disease in rheumatoid arthritis
starting penicillamine. The most likely diagnosis is include analgesic nephropathy, focal segmental
penicillamine-induced membranous nephropathy, which glomerulonephritis and rheumatoid vasculitis. All are
usually occurs within 6–12 months of the initiation of characterized by blood in the urine. Analgesic
drug therapy. Proteinuria resolves in virtually all cases nephropathy is usually secondary to non-steroidal anti-
after stopping the drug but this may take several months. inflammatory drugs and paracetamol. The proteinuria is
Other causes of heavy proteinuria secondary to rarely severe enough to cause nephrotic syndrome. Focal
membranous nephropathy in rheumatoid arthritis include segmental glomerulonephritis is rare and is excluded by
gold therapy. the absence of red cells in the urine. Rheumatoid
vasculitis has a predilection for skin and the peripheral
Renal amyloidosis is a recognized cause of heavy nervous system but in very rare circumstances may affect
proteinuria complicating chronic rheumatoid arthritis. the kidneys. It is more likely in patients with severe
While it is possible that the patient may have renal disease, nodule formation, high titres of rheumatoid
amyloidosis, the relationship of the proteinuria to the factor and hypocomplementaemia. (See Question 320.)

Answer 18

d. Metastatic spinal cord compression. vertebra in question is infiltrated by tumour and appears
white. The vertebra above are also infiltrated with
This is a T2 weighted image that shows evidence of cord tumour (appear white). The vertebra below the collapsed
compression from a collapsed thoracic vertebra. The vertebra appear normal (black).

Interpretation of MRI Scans

Substance T1 weighted T2 weighted

Water/vitreous/CSF black light grey or white
Fat white light grey
Muscle grey grey
Air black black
Fatty bone marrow white light grey
Brain white matter light grey grey
Brain grey matter grey very light grey

T1 Weighted Imaging T2 Weighted Imaging

Provides anatomical information Provides pathological information

Low signal – Black Low signal – Black
• Cortical bone • Cortical bone
• Air • Air
• Rapidly flowing blood • Rapidly flowing blood
• CSF • Haemosiderin

Intermediate signal – Grey Intermediate signal – Grey
• Grey matter is darker than white matter • White matter is darker than grey matter

High signal – White High signal – White
• Fat in bone, scalp and orbit • CSF or water

Clinical Cases 23

Question 19

A 16-year-old girl presented with intermittent episodes of of a BCG scar on inspection of the left upper arm. Both
lower colicky abdominal pain for six months. In the heart sounds were normal and the chest was clear.
interim she had lost almost 6.4 kg in weight. Her Abdominal examination revealed vague tenderness
appetite was not impaired. There was no history of affecting the hypogastrum and right iliac fossa.
diarrhoea, although the patient had complained of
intermittent constipation and abdominal bloating. The Investigations are shown.
patient was English in origin. She had no family history
of note. She had last travelled abroad to Barbados on Hb 10 g/dl
holiday a year ago. The only other past medical history WCC 11 ϫ 109/l
included a short episode of painful ankles associated with Platelets 498 ϫ 109/l
circular erythematous skin lesions. ESR 55 mm/h
U&E Normal
On examination she was thin and mildly clubbed. The AST 20 iu/l
heart rate was 90 beats/min and regular. The blood ALT 22 iu/l
pressure measured 100/55 mmHg. There was evidence Bilirubin 12 ␮mol/l
Albumin 33 g/l
What is the diagnosis? Stool culture Negative
a. Sarcoidosis. Chest X-ray Minor calcification, a few
b. Intestinal lymphoma.
c. Intestinal tuberculosis. perihilar nodes
d. Crohn’s disease.
e. Irritable bowel syndrome.

Question 20

A 24-year-old patient was admitted to hospital with acute The patient improved significantly over the next 48
asthma for the fourth time in the past six years. The hours but then suffered three successive grand mal
asthma was usually precipitated by a coryzal illness or seizures, which necessitated ventilation.
exposure to allergens. There was no other past medical
history of note. The patient usually inhaled ventolin as What was the most likely cause of the epileptic
required, salmeterol inhaler twice daily, becotide inhaler seizures?
twice daily and had recently been prescribed
aminophylline 450 mg twice daily. a. Hypoxia.
b. Meningitis.
On admission she had a bilateral wheeze. The PEFR c. Benign intracranial hypertension.
was 200 l/min. The oxygen saturation on air was 86% d. Theophylline toxicity.
and on 28% oxygen it was 94%. The chest X-ray revealed e. Herpes encephalitis.
hyperinflated lungs. The patient was commenced on
nebulized bronchodilators, prednisolone 30 mg daily and
amoxycillin. The following day she developed a rash
therefore the amoxycillin was substituted with
erythromycin.

24

Answer 19

d. Crohn’s disease. Although ileo-caecal TB may present in a similar fashion,
her race and the presence of a BCG scar is against the
Abdominal cramps, weight loss, erythema nodosum diagnosis. Sarcoidosis enteropathy has been reported but
(raised circular skin lesions) and raised inflammatory this is very rare and usually in association with other
markers are highly suggestive of inflammatory bowel features of this multi-system disorder. Small bowel
disease. Tenderness in the right iliac fossa points to the lymphoma may present in a similar fashion; however,
possibility of terminal ileal disease and hence Crohn’s diarrhoea is a prominent feature. Raised inflammatory
disease, although this is a non-specific feature since many markers are against the diagnosis of irritable bowel
conditions may cause right iliac fossa tenderness. disease, which is a functional rather than inflammatory
Diarrhoea is not always a prominent feature in Crohn’s disorder. (See Answers 31, 394.)
disease.

Answer 20

d. Theophylline toxicity. of asthma, hypokalaemia (sometimes a consequence of
nebulized salbutamol) is also associated with theophylline
The question tests the candidate’s knowledge about toxicity.
drugs interacting with aminophylline and inhibiting its
metabolism. With respect to the treatment of lower Symptoms do not usually occur until plasma
respiratory tract infections, both quinolone and theophylline concentrations exceed 20 mg/l. The most
macrolide antibiotics (e.g. ciprofloxacin, erythromycin adverse effects of theophylline toxicity, such as cardiac
respectively) inhibit aminophylline metabolism. arrhythmias and seizures, generally occur at plasma
theophylline levels >40 mg/l.
Features of theophylline toxicity include nausea,
vomiting, hypotension, cardiac arrhythmias and seizures. The management of theophylline toxicity is usually
Other drugs that inhibit the metabolism of theophylline supportive. In patients who have taken an overdose, the
include cimetidine, propranolol, allopurinol, aim is to prevent absorption in the stomach. There are
thiobendazole and the contraceptive pill. In the context three main strategies in the management of theophylline
toxicity (shown below):

Strategy 1 (if patient is stable)
• Gastric lavage followed by oral activated charcoal administration is effective.

Strategy 2
• Treat arrhythmias with beta-blockers; unfortunately many patients taking theophylline for therapeutic reasons

have contraindications to beta-blockers. In these patients lignocaine may be used for ventricular arrhythmias
and verapamil for supraventricular arrhythmias including atrial fibrillation.
• Treat seizures with diazepam or barbiturates; phenytoin is not very effective.

Strategy 3 (rarely required)
• Haemodialysis is very effective in treating life-threatening toxicity, i.e. patients with a plasma theophylline

level of >100 mg/l who have profound hypotension, fatal cardiac arrhythmias and seizures. Age and
concomitant hepatic disease are important factors in relation to prognosis with theophylline toxicity. Patients
aged >60 years with liver disease may be dialysed at theophylline levels of around 60 mg/l.

Clinical Cases 25

Question 21

A 64-year-old Asian man presented with a six-week
history of dyspnoea and wheeze. For two weeks he had
also developed a cough productive of yellow sputum and
fever. There was no history of night sweats. The patient
had not travelled abroad for over 20 years.

Investigations are shown.

Hb 13 g/dl What is the most probable diagnosis?
WCC 11 ϫ 109/l a. Churg–Strauss syndrome.
(neutrophils 8 ϫ 109/l, b. Tuberculosis.
Platelets lymphocytes 1 ϫ 10 9/l c. Allergic bronchopulmonary aspergillosis.
ESR eosinophils 2 ϫ 109/l) d. Tropical pulmonary eosinophilia.
Biochemistry 258 ϫ 109/l e. Asthma.
Chest X-ray 30 mm/h
Sputum culture
Normal

Diffuse perihilar infiltrates

Negative

Question 22

A 78-year-old patient presented with sudden onset of fibrillation with a rapid ventricular rate and q waves in the
severe breathlessness. He had a history of ischaemic heart anterior leads.
disease and had suffered two myocardial infarctions in the
past three years. He had an 11-year history of What is the safest drug for restoring sinus rhythm?
hypertension that had been well controlled. He was a a. IV digoxin.
non-smoker. His medication consisted of aspirin, b. IV amiodarone.
ramipril, atenolol, bendroflumethiazide and simvastatin. c. IV flecanide.
d. IV esmolol.
On examination he had a heart rate of 146 beats/min. e. IV dofetolide.
The pulse was irregular. The blood pressure was
100/68 mmHg. Both heart sounds were quiet.
Auscultation of the lungs revealed widespread inspiratory
crackle and expiratory wheeze. The ECG showed atrial

Question 23

An 81-year-old man with non-insulin-dependent diabetes What is the best combination of infusions in the
mellitus was found unconscious by his carer. Blood tests management of the patient?
performed on admission to hospital are shown.
a. IV saline (0.45%), IV insulin and subcutaneous
Sodium 153 mmol/l heparin.
Potassium 5.4 mmol/l
Urea 40 mmol/l b. IV saline (0.9%), IV insulin and subcutaneous
Creatinine 310 ␮mol/l heparin.
Glucose 60 mmol/l
c. IV sodium bicarbonate, IV insulin and
subcutaneous heparin.

d. IV dextrose saline, IV insulin and subcutaneous
heparin.

e. IV dextrose (5%), IV insulin and subcutaneous
heparin.

26

Answer 21

c. Allergic bronchopulmonary aspergillosis. bronchiectasis and parenchymal infiltrates in the perihilar
area. The presence of high titres of IgE and IgG
The history of cough sputum, eosinophilia and perihilar antibodies and a positive hypersensitivity skin test to
infiltrates is most consistent with allergic broncho- Aspergillus fumigatus testing confirm the diagnosis.
pulmonary aspergillosis in the context of the history Treatment is with a prolonged course of itraconazole.
given. There is no drug history to indicate an
eosinophilic pneumonitis, nor a history of travel to the Tropical pulmonary eosinophila is an immune reaction
tropics to suggest tropical pulmonary eosinophilia. to infection with the human filarial parasites Wucheria
Churg–Strauss syndrome is unlikely in the absence of bancrofti and Brugia malayi. It is characterized by a non-
vasculitis, neuropathy or renal involvement. Asthma does productive cough, wheeze, fever, weight loss,
not cause pulmonary infiltrates. Tuberculosis does not lymphadenopathy, eosinophilia and patchy infiltrates on
usually cause eosinophilia. the chest X-ray. The condition occurs in patients infected
in the tropics. The worm is rarely identified but the
The diagnosis of allergic bronchopulmonary condition responds to diethycarbamazine, the drug
eosinophilia is made in patients with asthma, proximal normally used to treat filariasis. (See Question 396.)

Answer 22

b. IV amiodarone. anti-arrhythmic agent that is effective at restoring sinus
rhythm in patients with persistent AF (up to seven days)
The patient has rapid atrial fibrillation in the context of or more permanent AF. Dofetolide is less negatively
ischaemic heart disease and has evidence of pulmonary inotropic than many other drugs that may be effective in
oedema. One has to assume that left ventricular function restoring sinus rhythm such as propafenone (class IC),
is impaired to answer this question since it is highly quinidine, disopyramide (class IA) and sotalol (class III),
unlikely that a heart rate of 146 beats/min would cause but experience regarding its use in the UK is relatively
left ventricular failure in a patient with normal left limited. Digoxin is effective in controlling ventricular rate
ventricular function. Amiodarone, dofetolide and in AF but does not restore sinus rhythm. Esmolol is a
flecainide are capable of restoring sinus rhythm. Of these very short-acting beta-blocker (class II antiarrhythmic
amiodarone is the least negatively inotropic. agent) that is not useful at restoring sinus rhythm.

Flecainide is relatively contraindicated in patients with
known coronary artery disease. Dofetolide is a class III

Answer 23

b. IV saline (0.9%), IV insulin and subcutaneous the development of cerebral oedema. If despite adequate
heparin. hydration, the sodium remains >150 mmol/l some
authorities advocate switching to half-strength saline
The patient has a hyperosmolar non-ketotic diabetic (0.45%). The patient has severe dehydration creating a
coma (HONK). The fluid of choice is saline. The hyperviscosity state that may predispose him to arterial
strength of saline used initially is always 0.9% since it is and venous thromboses. Heparin therapy is mandatory to
effective at restoring volume and has a lower risk of prevent such complications during the management of
causing large drops in plasma osmolality, a risk factor for HONK. (See Question 84.)

Clinical Cases 27

Question 24 1: What is the most probable diagnosis?
a. Haemolytic–uraemic syndrome.
A 13-year-old girl was admitted with a two-day history of b. IgA nephritis.
lower abdominal pain and blood-stained diarrhoea. Three c. Henoch–Schönlein purpura.
days later, she developed pains in her ankles and right d. Systemic lupus erythematosus.
elbow and felt nauseous. Positive findings on e. Polyarteritis nodosa.
examination were a purpuric rash affecting the arms and
legs, periorbital oedema and a blood pressure of
150/95 mmHg.

Investigations are shown.

Hb 10 g/dl 2: List two investigations that would be most useful
WCC 12 ϫ 109/l in confirming the diagnosis.
Platelets 136 ϫ 109/l a. Skin biopsy.
MCV 70 fl b. Renal biopsy.
ESR 35 mm/h c. Blood cultures.
PT 13 s (control 13 s) d. TT.
APTT 34 s (control 36 s) e. Serum fibrinogen level.
Sodium 138 mmol/l f. Serum IgA level.
Potassium 5.9 mmol/l g. Serum ANF level.
Creatinine 130 ␮mol/l h. Serum ANCA.
Urinalysis Blood ++ i. Blood film.
Protein ++ j. 24-hour urine collection for protein.

Question 25

A 39-year-old African male was referred to the blood
pressure unit with persistent blood pressure readings of
140–150/90–95 mmHg over the past six months. He
was a non-smoker and consumed 4 units of alcohol per
week. The patient weighed 89 kg and measured 1.7 m.
Physical examination was normal with the exception of a
blood pressure reading of 150/92 mmHg.

Investigations are shown.

Sodium 136 mmol/l
Potassium
Urea 4.2 mmol/l
Glucose
Total cholesterol 5 mmol/l
Triglycerides
12-lead ECG 4.1 mmol/l

Urinalysis 4.1 mmol/l What is the best initial management for the raised
blood pressure?
1.2 mmol/l
a. Beta-blocker.
Sinus rhythm b. Angiotensin-converting enzyme inhibitor.
c. Low-salt diet, regular exercise.
Right axis deviation d. Calcium channel antagonist.
e. Thiazide diuretic.
Protein 0

Blood 0

Cells 0

28

Answer 24

1: c. Henoch–Schönlein purpura. The diagnosis is usually clinical; however, tissue
2: a. Skin biopsy. diagnosis is possible with skin or renal biopsy. Skin
biopsy demonstrates a leucoclastic vasculitis with IgA
b. Renal biopsy. deposition. Renal biopsy reveals mesangial IgA
deposition associated with a glomerulonephritis. Renal
The combination of lower abdominal pain, bloody histology is indistinguishable from IgA nephropathy. In
diarrhoea, purpuric rash and nephritis in a young girl this case, more marks are given to skin biopsy because it
are highly suggestive of Henoch–Schönlein purpura. is safer and more practical than renal biopsy. Serum IgA
The condition is a small-vessel vasculitis that occurs levels are depressed in approximately 50% of cases. The
most commonly in children aged 4–15 years. It is management is usually supportive, although there may
characterized by gastrointestinal symptoms which be a role for methylprednisolone in cases of acute
comprise abdominal pain, diarrhoea, and rectal crescentic nephritis.
bleeding, flitting arthralgia affecting large joints, a
purpuric rash characteristically affecting the lower limbs The condition must not be confused with the
and buttocks and an acute nephritis. Complications haemolytic uraemic syndrome (discussed in Answer
include intestinal perforation, haemorrhage and 152), which is also characterized by diarrhoea and renal
intussusception and acute renal failure. failure.

Answer 25

c. Low-salt diet, regular exercise. Caribbean patients. However, these patients may
respond to ACE inhibitors and beta-blockers when
The patient is young and has mild hypertension on prescribed with drugs that activate the renin–
presentation. He does not have any other risk factors for angiotensin–aldosterone system, i.e. thiazide diuretics
cardiovascular disease or evidence of secondary end- and calcium channel blockers.
organ damage as a result of the raised blood pressure.
In this particular case the initial management plan Both lifestyle modification and pharmacological
should include a low-salt diet, regular exercise and therapy would be indicated if the patient had a blood
weight loss. The patient should be observed carefully pressure ≥160/100 mmHg, or evidence of secondary
for up to a year and should only be commenced on end-organ damage, or other risk factors for coronary
pharmacological therapy if the blood pressure remains artery disease at presentation.
above 140/85 mmHg.
There is a high prevalence of hypertension in
If treatment is indicated after a year, the drugs of individuals of Afro-Caribbean origin, with almost 50%
choice are thiazide diuretics or calcium channel of patients over the age of 40 years being affected. This
blockers. Angiotensin-converting enzyme inhibitors and particular group of patients generally develop hyper-
beta-blockers are not particularly effective as tension at a younger age and are at higher risk of
monotherapy because both drugs act by suppressing hypertensive complications such as stroke, heart failure
renin levels, which are already relatively low in Afro- and renal failure than Caucasian patients. Hypertension
in Afro-Caribbean patients is salt sensitive and responds
well to a low-salt diet.

Clinical Cases 29

Question 26

26a 26b

A 41-year-old male was admitted to hospital with acute On examination, he was confused. He had a wide-
confusion. He had been generally unwell for two days. A spread rash (26a). His left eye is shown (26b). The heart
worried neighbour looked through his letter box when he rate was 120 beats/min; the blood pressure was
failed to answer the doorbell, and found him lying on the unrecordable. There was no evidence of nuchal rigidity,
floor. There was no other history of note. and Kernig’s sign was negative. There was no focal
neurological deficit. Examination of the cardiovascular,
Hb 11 g/dl respiratory and gastrointestinal tract was normal.
WCC 24 ϫ 109/l
Platelets 30 ϫ 109/l Investigations are shown.
Sodium 135 mmol/l
Potassium Which single investigation would you perform next?
Creatinine 5 mmol/l a. Blood cultures.
PT 156 μmol/l b. CT scan brain.
Fibrinogen degradation 48 s (control 13 s) c. CSF for microscopy and culture.
d. Chest X-ray.
products >25,000 e. Throat swab for culture.

Question 27 Investigations are shown.

A 36-year-old woman is seen in the Accident and Sodium 138 mmol/l
Emergency Department after having taken 40 paracetamol Potassium 3.9 mmol/l
tablets with a quarter-bottle of vodka six hours earlier, Urea 5.1 mmol/l
following an argument with her husband. She was Bicarbonate 18 mmol/l
nauseous, but had not vomited. There was no past medical PT 15 s (control 15 s)
history of note. A physical examination was normal. Serum paracetamol 200 mg/l

1. List two immediate management steps from the level
following list:
2. Which one of the following investigations would
a. Gastric lavage. you perform the next day to assess her prognosis?
b. IV dextrose 5%.
c. Oral activated charcoal. a. Serum potassium.
d. Fresh-frozen plasma. b. Serum magnesium.
e. IV sodium bicarbonate. c. Arterial pH.
f. IV N-acetyl cysteine. d. Serum paracetamol concentration.
g. IV flumazenil. e. Serum aspartate transaminase.
h. IV vitamin K.
i. IV saline (0.9%).
j. IV prochlorperazine.

30

Answer 26

a. Blood cultures. the presence of very low levels of platelets and DIC is
contraindicated because of the dangers of bleeding into the
The eye demonstrates a conjunctival haemorrhage. The spinal canal, particularly because the yield is just as high
rash is a necrotic purpuric rash, which is typical of from nasal swabs. It is also possible to isolate the
meningococcal septicaemia. The patient has septic shock meningococcal antigen from blood before blood culture
and requires immediate therapy. The recognition that he results are available. This test is particularly useful if
has meningococcal septicaemia is important for the choice antibiotics have been given before the patient is brought to
of antibiotics that you will use. In medical emergencies, the hospital (negative blood cultures).
reader must be familiar with the drugs that are used in that
particular emergency, but not necessarily the dosage, as Meningococcal meningitis and septicaemia are caused
this can be found in the British National Formulary or the by serogroups B and C. Septicaemia is associated with
equivalent. Although most Neisseria meningitidis strains widespread petechial haemorrhage. Conjunctival
are sensitive to benzyl penicillin, it is prudent to cover the haemorrhage may be the first physical manifestation. Shock
patient with additional cephalosporin or aminoglycoside is common owing to the production of a circulating
antibiotic therapy until the sensitivities of the organism are endotoxin. DIC is a commonly recognized complication
known. The circulation must be restored to prevent that may result in adrenal haemorrhage
hypoperfusion of vital organs, particularly the kidneys. The (Waterhouse–Friderichsen syndrome). Meningitis is often
presence of low platelets, high fibrinogen degradation characterized by a myalgia, headache, photophobia, neck
products and abnormal clotting is suggestive of DIC, stiffness, nausea and vomiting. In the absence of DIC, the
which should be treated with fresh-frozen plasma to diagnosis is made rapidly by performing a Gram stain on
prevent haemorrhage. the CSF. Blood cultures are positive in the majority of
patients with meningitis. Focal neurological signs are less
Neisseria meningitidis, the causal Gram-negative common than in pneumococcal meningitis.
diplococcus, can be cultured from the CSF in over 80% of
cases with evidence of neurological involvement. However, Note: individuals in contact with affected patients must
receive rifampicin chemoprophylaxis.

Answer 27

1. c. Oral activated charcoal. cellular glutathione. The liver is unable to deactivate
f. IV N-acetyl cysteine. NAPQI, which is responsible for massive hepatic necrosis
and hepatic failure. Patients may have nausea, anorexia or
2. c. Arterial pH. vomiting on the first day. After 72 hours, features of liver
and renal failure may ensue.
The drug should be given within 8–10 hours of ingestion
of the overdose, and continued while the liver function is The three most important prognostic markers in
abnormal. It is useful because it replenishes cellular paracetamol overdose are serum creatinine concentration,
glutathione stores and reduces oxidative damage caused arterial pH and prothrombin time. A rise in serum
by the toxic metabolite, NAPQI. An alternative to this is creatinine level due to renal failure is a bad prognostic
methionine. Gastric lavage is useful if performed within 1 sign. A level of over 300 mmol/l is associated with over
hour of the overdose. 70% mortality. Systemic acidosis (due to the failure of
clearance of lactate by the liver) more than 24 hours after
The patient has taken 20 g of paracetamol. An the overdose is associated with a poor prognosis. A pH of
ingestion of 15 g is considered potentially serious in most below 7.3 is associated with only a 15% chance of survival.
patients. The toxicity of paracetamol is related to the The PT is usually the first liver test to become abnormal.
production of a toxic metabolite of paracetamol. This is A PT of >20 s at 24 hours after overdose is suggestive of
NAPQI, which usually is immediately conjugated with significant hepatic damage, and a peak PT of >180 s is
glutathione and excreted. In paracetamol overdose, the associated with a 90% mortality.
toxic metabolite is produced in excess and depletes

Clinical Cases 31

Question 28 28

A 52-year-old female was brought Hb 14 g/dl
into the Accident and Emergency WCC 12 ϫ 109/l
Department after being found Platelets 221 ϫ 109/l
collapsed outside a public house. MCV 88 fl
There was no one accompanying Sodium 144 mmol/l
her, and there was no information Potassium 4.0 mmol/l
regarding her next of kin. Urea 13 mmol/l
Creatinine 100 μmol/l
On examination, she was very Bicarbonate 20 mmol/l
drowsy and had a Glasgow coma Chloride 108 mmol/l
score of 6 out of 15. Her pupils Calcium 2.4 mmol/l
were 10 mm each and reacted very Phosphate 1.3 mmol/l
sluggishly to light. On attempting to Bilirubin 12 μmol/l
examine her fundi, she was noted to AST 33 iu/l
have coarse nystagmus, but a clear Gamma GT 28 iu/l
view of her fundi did not Alkaline phosphatase 120 iu/l
demonstrate any abnormalities. The tone in all her limbs Plasma osmolality 333 mOsm/l
was increased and her reflexes were brisk. The plantars Urine osmolality 120 mOsm/l
were both upgoing. The heart rate was 135 beats/min, Blood glucose 6 mmol/l
and regular. The blood pressure was 105/60 mmHg. The Chest X-ray Normal-sized heart and
respiratory rate was 20/min. Examination of the
precordium and lung fields was normal, but examination of Skull X-ray clear lung fields
the abdomen revealed a firm palpable mass 4 cm above the Brain CT scan Normal; no fractures seen
symphysis pubis. The patient was catheterized and drained Lumbar puncture: Normal
of 2 litres of urine. Investigations are shown.
CSF pressure 100 mmH2O
Shortly after the lumbar puncture, the patient had a Cells 3/mm3
generalized seizure which lasted 30 s. The attending nurse Protein 0.35 g/l
raised concerns about an arrhythmia on the cardiac Glucose 3 mmol/l
monitor, and a 12-lead ECG was performed (28). ECG Sinus tachycardia; right

1. Calculate the plasma osmolality. axis deviation
2. Explain the discrepancy between the calculated

plasma osmolality and the measured plasma
osmolality.
3. Give two possible explanations for the low urine
osmolality.
4. What is shown on the ECG?
5. What diagnosis best fits all the information given
above?
6. What three investigations would you perform to
help in this patient’s management?

32

Answer 28

1. The plasma osmolality is calculated by the formula 3. Nephrogenic diabetes insipidus from lithium
2 ([Na] + [K]) + [Urea] + [Glucose]. In this case, therapy or inhibition of ADH secretion as a result
the calculated plasma osmolality is 315 mOsm/l. of alcohol ingestion.

2. The measured plasma osmolality is higher than the 4. There is a broad-complex tachycardia with extreme
calculated one, suggesting that the patient has axis deviation and concordance of the QRS
ingested something which has not been measured, complexes in the chest leads. These findings are
but has the effect of increasing the plasma suggestive of ventricular tachycardia.
osmolality. The most likely possibility in this case is
alcohol ingestion. Although lithium contributes to 5. Tricyclic antidepressant drug overdose with
plasma osmolality, it would be very unusual for the alcohol.
lithium concentration to be high enough to
increase the plasma osmolality by 18 mOsm/l, 6. i. Arterial blood gases.
considering that a serum lithium concentration of ii. Serum lithium level.
2.5 mmol/l causes dangerous toxicity. iii. Blood alcohol level.

Arrhythmias usually settle on correction of hypoxia and anticholinergic effects are not a feature. In addition,
acidosis. Administration of class I antiarrhythmic agents lithium toxicity is associated with ataxia and dysarthria.
may paradoxically worsen arrhythmias, with the exception Chronic lithium ingestion may cause hypothyroidism.
of phenytoin. Status epilepticus should be corrected with Sodium-depleting drugs such as diuretics lead to excess
intravenous diazepam. absorption of lithium by the kidney, and predispose to
toxicity.
Epileptic seizures and ventricular arrhythmias in a
patient found collapsed should raise the suspicion of The arterial blood gases are an important investigation
tricyclic antidepressant drug overdose. The low urine because they will identify hypoxia and acidosis, both of
osmolality suggests that the patient has probably taken which precipitate ventricular arrhythmias in patients with
the overdose together with alcohol, and it is possible that tricyclic antidepressant overdose. The serum lithium level
she may have also taken lithium. The normal CT scan of will be useful to determine whether lithium has been
the brain and normal CSF are against pathology in the ingested, and will help decide whether the patient should
central nervous system. She has dilated pupils, which is have forced diuresis. In general, patients with a serum
against narcotic abuse. Indeed, the combination of lithium of >3 mmol/l should have forced diuresis.
dilated pupils, tachycardia and urinary retention are all Haemodialysis is recommended if serum lithium exceeds
suggestive of the anticholinergic side-effects of tricyclic 4 mmol/l.
antidepressants. Severe lithium toxicity is associated with
seizures, coma and ventricular arrhythmias, but The management of the patient is outlined below.

The management of tricyclic antidepressant overdose

• Protect the airway, and give oxygen via a mask
• Gastric lavage under anaesthetic supervision (within 12

hours of ingestion) followed by activated charcoal via a
nasogastric tube
• Monitor on a high-dependency unit
• Correct hypoxia
• Correct acidosis with IV sodium bicarbonate
• Intravenous fluids to improve blood pressure
• Epileptic seizures should be corrected with IV lorazepam
or diazepam. Phenytoin is contra-indicated
• Ventricular arrhythmias respond to correction of acidosis
and hypoxia. IV sodium bicarbonate is the mainstay of
prevention and treatment of ventricular arrhythmias and
should be administered in all patients with ventricular
tachycardia or acidosis or in patients with a QRS duration
>110 msec

Clinical Cases 33

Question 29

29a 29b

A 76-year-old male presented with a three-month history 29c
of anorexia, weight loss and fever. Apart from sweating
excessively at night and feeling very thirsty, he did not
have any other symptoms. He was a non-smoker and had
been a schoolteacher for 40 years before retiring.

On examination, he was thin. The finding on
inspection of his hands is shown (29a). The heart rate
was 100 beats/min and the blood pressure 180/105
mmHg. His temperature was 37.8°C (100°F).
Examination of the cardiovascular system and the
respiratory system was normal. Abdominal examination
revealed minimal tenderness and some fullness in the
right loin. Examination of the genitalia revealed some
oedema of the scrotum. The lower limbs were
oedematous.

Investigations are shown.

Hb 18 g/dl

WCC 10 ϫ 109/l

Platelets 300 ϫ 109/l

ESR 110 mm/h

Sodium 140 mmol/l

Potassium 3.1 mmol/l

Creatinine 120 μmol/l

Calcium 2.6 mmol/l

Albumin 36 g/l

Chest X-ray (29b)

CT scan of abdomen (29c)

Urinalysis Blood +++

Protein +

Bilirubin 0

1. Give two explanations for the serum calcium level.
2. What is the most probable diagnosis?
3. List three important tests you would perform to

help achieve a diagnosis.
4. What is the management?

34

Answer 29

1. i. Bone metastases. months before the diagnosis is made. The neoplastic cells
ii. PTH-related peptide secretion from a right-sided often produce peptide hormones such as erythropoietin,
renal carcinoma. renin, ADH and PTH-related peptide. This patient has a
relative polycythaemia, hypercalcaemia, hypokalaemia
2. Right-sided renal carcinoma. and hypertension, which reflect erythropoietin, PTH-
3. i. Renal angiography. related peptide and renin secretion, respectively. Fever is
present in approximately 20% of patients and is probably
ii. Renal venography and inferior vena cavogram. secondary to the secretion of a pyrogen by the tumour.
iii. Bone scan to detect bony metastases. Hypertension is present in approximately 30% of patients.
4. Surgical removal of the right kidney if his general Metastases usually occur via the bloodstream, although
direct invasion of the renal veins or the inferior vena cava
health will allow. is relatively common. Some 10% of hypernephromas are
bilateral, so close attention is given to the contralateral
This patient presents with anorexia, weight loss, and a kidney when reviewing the CT scan. Venography and
fever that may represent sepsis or malignancy. The right arteriography allow assessment of invasion of the veins
loin tenderness and haematuria are suggestive of renal and the vascularity of the tumour, respectively. Urine
involvement. The chest X-ray reveals multiple opacities in cytology may reveal malignant cells but the diagnostic
both lungs which represents a cannon-ball metastases yield is low. Removal of the hypernephroma (even when
from the right kidney. CT scan of the abdomen reveals a distant metastases are present) improves survival and
carcinoma of the right kidney which is invading the causes regression of the metastases in many, but not all,
inferior vena cava; hence the scrotal and lower-limb patients. Radiotherapy and chemotherapy have been used
oedema. Hypernephroma characteristically presents with in the treatment of this tumour, but the results are not
a triad of haematuria, loin pain and swelling. Haematuria very encouraging. The overall survival rate is 30–50%.
is present in 50% of cases, but pain and swelling are less
frequent. Non-specific symptoms such as anorexia,
weight loss and fatigue may be present for several

Question 30

An 84-year-old female was referred to clinic with increasing movements of her arms and legs. There was also clinical
forgetfulness. Her GP had commenced her on a small dose evidence of increased tone and cogwheel rigidity on
of haloperidol for agitation eight months ago. According to neurological examination of her limbs.
the staff at the nursing home where she resided, she had
become increasingly confused over the past few months A CT scan of the brain revealed generalized cerebral
and more recently had developed odd movements affecting atrophy and calcification of the basal ganglia.
her face, arms and legs. Her GP had reviewed her two
weeks previously and stopped the haloperidol; however, she What is the cause of her movement disorder?
remained confused and the movement disorder had a. Multi-infarct dementia.
become much more pronounced. She was not taking any b. Lewy body dementia.
other medication. c. Extrapyramidal side-effects of haloperidol.
d. Pseudohypoparathyroidism.
On examination, she had a mental test score of 4/10. e. Hypoparathyroidism.
Her vital parameters were normal. She exhibited
intermittent yawning motions of the mouth, with
occasional tongue protrusion. There were semi-purposeful

Question 31

A 33-year-old Iranian male was investigated for a six- diarrhoea which resolved after a week. He was seen by a
month history of general malaise, weight loss, fever, pain gastroenterologist shortly afterwards, who diagnosed an
in his knees, ankles and wrists and a sore mouth. On inflammatory colitis, possibly secondary to infection. A
systematic enquiry, he gave a two-year history of a rectal biopsy was performed by the gastroenterologist, and
recurrent sore mouth that made it difficult for him to eat. this was reported as a non-specific colitis. The patient had
Just before the onset of all his symptoms he had never experienced any abdominal symptoms after this, but
experienced an attack of abdominal pain and bloody had several episodes of soreness affecting the mouth. In

Clinical Cases 35

addition, he developed painful eyes and pain on Examination of his oral cavity revealed an abnormality
intercourse and on voiding urine. There was no history of (31a). His eyes were sore (31b). He had submandibular
urethral discharge. Shortly afterwards he was admitted to lymphadenopathy. His ankle, wrist and knee joints were
hospital with a femoral vein thrombosis, which was treated tender, and joint movements were restricted. In addition,
with anticoagulants and thought to be secondary to he had painful lesions on his legs (31c). Examination of
dehydration and immobility from his diarrhoeal illness. his genitalia and anal areas are shown (31d, e). He also
During the past six weeks his health had deteriorated. He pointed out an erythematous lesion approximately 2 cm in
had arthralgia and a fever. He had been married for five diameter that had developed at the site of venepuncture
years. He denied extramarital sex. His wife was well, and during a blood test performed by his GP two days earlier.
had not experienced any similar symptoms. All other aspects of the physical examination were normal.
Investigations are shown.
On examination, the patient appeared unwell.

31a 31b

31c 31d

31e

Hb 9.8 g/dl
WCC 13 ϫ 109/l
Platelets 450 ϫ 109/l
MCV 82 fl
CRP 200 g/l
Rheumatoid factor Absent
Antinuclear antibody Absent
Radiology of painful joints Normal

What is the most probable diagnosis?
a. Crohn’s disease.
b. Ulcerative colitis.
c. Reiter’s syndrome.
d. Behçet’s syndrome.
e. Gonococcal septicaemia.

36

Answer 30

c. Extrapyramidal side-effects of haloperidol. 30

The woman has clinical evidence of dyskinesia and neurological signs which include parkinsonism, chorea,
parkinsonism. The most common cause of her neurological epilepsy, ataxia and dementia. There is an association
signs is drug-induced extrapyramidal disease. Neuroleptic between calcification of the basal ganglia and
drugs which include haloperidol are extensively used in hypoparathyroidism or pseudohypoparathyroidism. Rare
treating agitation in the elderly. By blocking dopamine causes of basal ganglia calcification (30, arrowed) are
receptors in the basal ganglia, these drugs can offset cerebral irradiation and mitochondrial diseases.
extrapyramidal side-effects which include tremor, dystonia,
akathisia, parkinsonism and tardive dyskinesia. Acute
dystonic reactions appear within the first few hours or days,
and consist of oculogyric crises, torticollis or trismus.
Fortunately, they are uncommon, and resolve as soon as the
drug is withdrawn. Chronic tardive dyskinesias are the most
serious complication and affect 20% of patients on chronic
neuroleptic therapy. They usually occur after a patient has
been on treatment for at least three months, and can be
made worse in the first few weeks after stopping the
offending drug. In 60% of cases the dyskinesia resolves over
three years after drug withdrawal; however, in the
remainder of patients the movement disorder persists and is
very difficult to treat. Characteristic features involve lip
smacking, tongue protrusion, orofacial mouthing, trunk
rocking and distal chorea of the hands and feet. A
combination of any of these features may be present.

Cerebral calcification is an incidental finding in 0.5% of
CT scans in the elderly. About 20–30% of patients with
widespread calcification of the basal ganglia exhibit

Answer 31

d. Behçet’s syndrome. affects the knee and ankle joints. The most common joint
to be affected in the upper limb is the wrist, as in this case;
The patient has oral, genital and anal ulcers (31a, d and e, however, joint involvement occurs early, whereas in this
respectively), conjunctivitis (31b), arthritis, erythema case the disease has been present for two years.
nodosum (31c) and features of a systemic illness. There has
been a single episode of bloody diarrhoea and a previous Crohn’s disease is a chronic granulomatous
femoral vein thrombosis. The differential diagnoses include inflammatory disease of the gastrointestinal tract of
Crohn’s disease, Reiter’s syndrome and Behçet’s syndrome unknown cause, and is a strong possibility in this case.
(Table A). Reiter’s syndrome is classically a triad of Bloody diarrhoea is a recognized feature of Crohn’s colitis.
conjunctivitis, urethritis and arthritis 1–4 weeks after an Colonic disease is associated with perianal disease in just
episode of bacterial dysentery or a sexually transmitted over 30% of patients. A seronegative reactive arthritis is a
urethritis. Other features include plantar fasciitis, Achilles recognized complication of Crohn’s disease. Erythema
tendinitis, keratoderma blennorrhagica, circinate balanitis, nodosum occurs in some cases. Genital ulcers are rare, as is
stomatitis, hepatitis, cardiac and neurological involvement, deep-vein thrombosis. Urethral involvement and dysuria
and occasionally amyloidosis. It is possible that the only occur when an inflammatory fistula develops between
diarrhoeal illness may have been dysenteric and offset the the colon and the ureter. Recurrent urinary tract infections
reactive features of Reiter’s disease. There is no history of due to faecaluria can cause urethral stricture.
promiscuity or urethral discharge. Venous thrombosis
affects about 4% of patients with Reiter’s disease and occurs Behçet’s syndrome is the most probable diagnosis.
early in the disease. Mouth ulcers are common in Reiter’s Behçet’s syndrome is a recurrent multifocal disorder that
disease and are painless. Erythema nodosum is not a feature persists over many years. It is characterized by recurrent
of Reiter’s syndrome. Arthritis is asymmetrical and usually mouth and genital ulcers, ocular lesions, and skin, joint
and neurological involvement. The incidence is high in
Japan and in countries bordering the Mediterranean.
Oral and genital ulcers are present in most patients.

Clinical Cases 37

Ulcers can affect the pharynx and cause dysphagia. proteins are elevated and immune complexes are present.
Genital ulcers can cause dysuria and dyspareunia. Ocular The pathergy test is a simple useful test. Genital ulcers
lesions are the most serious development. Recurrent and oral complications are treated with topical steroids.
uveitis and iridocyclitis, retinal vascular lesions and optic In severe cases, systemic steroids become necessary,
atrophy can lead to loss of vision in 50% of patients with together with azathioprine, which acts as a steroid-
ocular involvement. Erythema nodosum is a recognized sparing agent. Colchicine, cyclosporin and levamisole
feature. Other skin manifestations include a diffuse have also been used in the management of this condition.
pustular rash affecting the face, erythema multiforme. The causes of orogenital ulceration are given in Table B.
The pathergy is a useful diagnostic sign. Pricking the skin
can lead to erythema around the affected part within Table A Causes of orogenital ulcers
24–48 hours, which is a relevant feature in our patient.
Behçet’s syndrome Syphilis
A seronegative arthritis affects about 40% of patients and Crohn’s disease Gonococcal infection
commonly involves knees, ankles and wrists. Recurrent Herpes simplex virus HIV
thrombophlebitis of the legs is a significant feature of Ulcerative colitis Pemphigus pemphigoid
Behçet’s syndrome, leading to venous thrombosis. Less Reiter’s syndrome Stevens–Johnson
often, superior or inferior cava thrombosis may occur. Lichen planus
Abdominal pain and bloody diarrhoea have also been syndrome
documented. Asymptomatic proteinuria is a recognized
feature, but on a few occasions may reflect renal Table B Causes of orogenital ulcers and venous
amyloidosis. Neurological complications occur in 20% of thromboses
patients. Organic confusional states, meningoencephalitis,
transient or persistent brainstem syndromes, multiple Behçet’s syndrome Ulcerative colitis
sclerosis and parkinsonian-type disorders are all recognized. Crohn’s disease Reiter’s syndrome

Behçet’s syndrome is a clinical diagnosis. There is no
specific diagnostic test. HLA-B51, B12, DR2, DR7 and
Drw52 are associated with the syndrome. Acute-phase

Question 32 32a
32b
A 16-year-old female was admitted with a six-month
history of myalgia, loss of weight and night sweats. Over
the past six weeks she had started to become breathless on
exertion. On admission to hospital she had a temperature
of 38.1°C (100.6°F). On auscultation of the precordium,
there was an early diastolic murmur at the left lower
sternal edge. Examination of the chest, abdomen and
central nervous system was normal, with the exception of
her fundi, one of which is shown (32a). An echocardio-
gram was performed to investigate the murmur (32b).

1. What is the diagnosis?
a. Systemic lupus erythematosus.
b. Polyarteritis nodosa.
c. Infective endocarditis.
d. Marantic endocarditis.
e. Libmann–Sacks endocarditis.

2. What investigation would you perform to confirm
the diagnosis?
a. Autoantibody screen.
b. Serum ANCA.
c. Serum complement.
d. Blood cultures.
e. Serology for atypical bacteria.

38

Answer 32

1. c. Infective endocarditis. vegetations <3 mm will not be seen on a transthoracic
2. d. Blood cultures. echocardiogram. A transoesophageal echocardiogram has
a much higher sensitivity in identifying bacterial
Fever, diastolic murmur, presence of echogenic mass on vegetations compared to transthoracic echocardiography.
the aortic valve and a Roth’s spot in the retina In approximately 10% of cases an organism is not
(haemorrhagic areas with a pale centre) are consistent identified despite repeated blood cultures. The two most
with the diagnosis of infective endocarditis. Clinical common reasons for this are prior treatment with
features of infective endocarditis are tabulated (Table A). antibiotics before culturing the blood, or that the
Blood cultures are the single most important endocarditis is due to a fastidious organism with respect
investigation in the diagnosis of infective endocarditis, to conventional blood culture media. In patients who are
and are positive in 90% of cases. Serial blood cultures immunosuppressed, endocarditis may be due to fungi
should be performed, because a single set of blood that may be difficult to culture, and in very rare
cultures may not necessarily reveal the culprit organism. circumstances endocarditis may be a manifestation of
Although transthoracic echocardiography is extremely SLE (Libmann–Sacks endocarditis) or a manifestation of
useful in confirming the presence of vegetations, the malignancy, when it is termed marantic endocarditis
absence of vegetations does not exclude endocarditis, as (Tables B and C).

Table A Causes of Roth spots and other stigmata of endocarditis

Causes of Roth spots Clinical features of endocarditis
Infective endocarditis Pyrexia
SLE Cardiac murmur
Polyarteritis nodosa Splinter haemorrhages
Severe anaemia Osler’s nodes: small; painful
Leukaemia Janeway lesions: macular; painless
Roth spots
Splenomegaly
Microscopic haematuria/nephritis

Table B Causes of culture-negative endocarditis Table C Common causes of culture-
positive endocarditis
Infective Non-infective
• Brucella spp. • SLE (Libmann–Sacs) • Streptococcus viridans
• Coxiella burnetti • Marantic endocarditis • Staphyloccus aureus
• Chlamydia spp. • Streptococcus faecalis (associated with
• Mycoplasma spp.
• Legionella spp. carcinoma of the colon)
• Histoplasmosis • Enterococci
• Tropherema whipelli
• Fungi

Clinical Cases 39

Question 33 33

A 51-year-old accountant presented with a six-month AST 60 iu/l
history of persistent dull right upper quadrant pain and Alkaline phosphatase 360 iu/l
fever. The pain did not radiate elsewhere, but was Bilirubin 82 μmol/l
exacerbated on lying on her right side. During this period Albumin 25 g/l
she had intermittent pale bulky stool which was difficult Total protein 93 g/l
to flush, and episodic dark urine. More recently, her INR 1.49
appetite was reduced and she had lost approximately 1 kg Chest X-ray Normal
in weight during the past month. Over the past week she
had difficulty sleeping due to itching all over her body, 1. What investigation would you perform to ascertain
and her colleague at work commented on a yellowish the cause of her illness?
pigmentation in her eyes. Six months before this, she had a. Blood cultures.
been relatively well. She had a past history of a b. Liver biopsy.
cholecystectomy for cholesterol stones at the age of 32 c. ERCP.
and subsequently had an ERCP and removal of sludge d. Ultrasound of the biliary tract.
from the common bile duct six years ago. She consumed e. Alpha-fetoprotein level.
10 units of alcohol per week. She was married with two
sons, aged 20 and 18. Three months ago she had been 2. What is the cause of her current presentation?
on holiday in Scotland. She was not taking any regular a. Sclerosing cholangitis.
medication. b. Secondary biliary cirrhosis.
c. Primary biliary cirrhosis.
On examination, she was slightly icteric. Inspection of d. Cholangiocarcinoma.
her hands is shown (33). e. Recurrent cholangitis.

There were spider naevi on her arms neck and face and
scratch marks around her trunk and lower limbs. She had
a temperature of 39°C (102.2°F). Her heart rate was
120 beats/min and blood pressure 140/80 mmHg.
There were a few inspiratory crackles on auscultation of
the right lung base. Abdominal examination demon-
strated firm, slightly tender hepatomegaly 4 cm below
the costal margin, and a moderately enlarged spleen.
There were no other abdominal masses, and there was no
evidence of shifting dullness. Rectal examination was
normal.

Investigations are shown.

Question 34 Arterial blood gases: 7.33
7 kPa
A 75-year-old male was seen by his GP with a five-day pH 6.5 kPa
history of wheeze and ankle swelling. He was prescribed 20 mmol/l
some medication, but continued to deteriorate and was PaO2 133 mmol/l
admitted to hospital. PaCO2 5 mmol/l
Bicarbonate 28 mmol/l
Investigations are shown. 200 μmol/l
Sodium
1. What is the acid-base disturbance?
2. Suggest two possible causes for this metabolic Potassium

picture. Urea

Creatinine

40

Answer 33

1. a. Blood cultures. ulcerative colitis, but may occur in patients with HIV
2. e. Recurrent cholangitis. either as a direct insult from HIV or due to opportunistic
CMV, cryptosporidial and Entamoeba infection. Although
The key in this question is a prior history of cholelithiasis it may lead to secondary biliary cirrhosis, it is unlikely in
affecting the gallbladder and the biliary tree. Stones in the this case, given the history of chronic cholestasis. Primary
bile duct are a nidus for infections, particularly when there biliary cirrhosis (PBC) is usually insidious. It is much more
has been instrumentation, as in this case. Cholangitis is common in females and usually presents with pruritus and
characterized by the triad of right upper quadrant pain, jaundice. Stones are not a recognized feature of PBC.
jaundice and fever. This patient has had previous episodes
suggestive of cholangitis, and the best answer for the The investigation of choice is blood cultures, which
presenting complaint is recurrent cholangitis. She also has have high positive yield in cholangitis. The most common
evidence of cirrhosis, which is almost certainly secondary to organism isolated is Escherichia coli. Hepatic
chronic cholestasis. Although cholangiocarcinoma may ultrasonography should be performed in all cases to
rarely complicate biliary cholestasis, this is unlikely given exclude a secondary hepatic abscess, and to assess the
the mode of presentation. Cholangiocarcinoma common bile duct. ERCP and sphincterotomy may be
characteristically presents with intermittent jaundice, and therapeutic. There is no role for α-fetoprotein and liver
causes duct obstruction before the onset of biliary cirrhosis. biopsy in this case. α-Fetoprotein is often modestly
Primary sclerosing cholangitis is very uncommon. The elevated in cirrhosis of the liver, but grossly elevated in
aetiology is thought to be auto-immune, but the trigger hepatocellular carcinoma. The presentation here is of
factor is unknown. More usually, it is associated with cholangitis. A liver biopsy should never be the first line of
investigation in a jaundiced patient with a fever.

Answer 34

1. Combined respiratory and metabolic acidosis. due to coexistent renal disease that prevents adequate
2.i. Acute cardiac failure. compensation, or to another factor causing metabolic
acidosis. The serum urea and creatinine levels are elevated,
ii. Severe exacerbation of obstructive airways disease but there is a relatively larger increase in the serum urea,
and pre-renal failure from diuretics. suggesting dehydration in this case. It is possible that he
was prescribed a diuretic for symptoms of chronic
The patient has a respiratory acidosis that is characterized obstructive airways disease, which have precipitated renal
by a PaCO2 of 6.5 kPa or more, and a pH below 7.35, and failure by causing dehydration.
is hypoxic. In the acute situation, respiratory acidosis is not
compensated by the kidney, but after 3–5 days the kidneys An alternative suggestion is that he has developed
retain bicarbonate ions to compensate, which results in severe cardiac failure leading to pulmonary oedema
normalization of the pH at the expense of a relative causing hypoxia and respiratory acidosis and
metabolic alkalosis. In this patient the bicarbonate is hypoperfusion of the kidneys, resulting in metabolic
slightly low, suggesting a metabolic acidosis. This may be acidosis from renal failure. Other causes of combined
respiratory and metabolic disorders are shown (Table).

Other causes of combined respiratory and metabolic acidosis

• Aspirin poisoning
• Severe pneumonia with renal failure due to septicaemia or interstitial nephritis (Legionnaire’s disease)
• Septicaemia from any cause complicated by ARDS
• Malaria complicated by pneumonia
• Acute renal failure and fluid overload
• Renal pulmonary syndromes: anti-GBM disease, Wegener’s granulomatosis, microscopic polyarteritis nodosa
• Acute massive pulmonary embolism
• Cardiac arrest (before ventilation)

Clinical Cases 41

Question 35

A 14-year-old male presented with joint pains, polydipsia
and polyuria.

Respiratory function tests are shown.

FEV1 (l) Actual Predicted 1. What is demonstrated by the respiratory function
FVC (l) 2.0 4.5 tests?
4.0 5.6
FEV1/FVC 50% 80% 2. In the context of the history, what is the diagnosis?
TLC (l) 4.9 7.0 3. How would you confirm the diagnosis?
1.7 2.2 4. If these were the respiratory function tests of a
Residual volume (l) 5.0 10
young male with recurrent respiratory tract
KCO (mmol/l/kPa) infections, what diagnosis would you consider?

Question 36 36a
36b
A 64-year-old patient who had been attending the
cardiology clinic for 15 years presented with increasing
breathlessness on exertion and a reduced appetite.
Examination of the face is shown (36a, b).

Investigations are shown.

Hb 13 g/dl
WCC 7 ϫ 109/l
Platelets 190 ϫ 109/l
MCV 80 fl
Sodium 135 mmol/l
Potassium 4.1 mmol/l
Urea 8 mmol/l
Creatinine 100 μmol/l
Bilirubin 48 μmol/l
AST 160 iu/l
Alkaline phosphatase 180 iu/l
Albumin 39 g/l
Thyroxine 190 nmol/l
TSH <0.1 mu/l
ECG Atrial fibrillation
Ventricular rate 80/min
Echocardiogram Enlarged left atrium
Normal-sized left ventricle

with good function
Normal valves

1. Give one possible explanation for the breathlessness
and all the abnormalities on the blood tests.

2. What investigation would you perform to
investigate the breathlessness further?

42

Answer 35

1. A mixed obstructive and restrictive defect with a common. Individual histiocytomas are pinkish-brown
reduced KCO. The low FEV1/FVC ratio is in papules of 1–5 mm in diameter. Respiratory mani-
keeping with an obstructive defect; however, the festations include lung fibrosis, bullae and large airway
low residual volume also suggests a restrictive obstruction by histiocytomas. Therefore, both restrictive
pattern. and/or obstructive respiratory function patterns may
occur. Bone involvement with marrow infiltration may be
2. The combination of polydipsia and polyuria with associated with pancytopenia. Any bone may be involved,
respiratory involvement is suggestive of diabetes but there appears to be a special predilection to the skull
insipidus. In a young patient with a mixed bones, where it manifests as large radiolucent lesions.
restrictive and obstructive defect the most likely Otitis externa and a troublesome aural discharge occur in
diagnosis is histiocytosis X. some patients. Ocular involvement is usually characterized
by a retro-orbital mass, resulting in proptosis. Hepato-
3. Transbronchial lung biopsy/liver biopsy/trephine splenomegaly and lymphadenopathy may be present.
aspirate may all show histiocytes and small round Occasionally, lymphadenopathy may be massive. One-
cells. third of patients develop diabetes insipidus due to
histiocytic infiltration of the pituitary stalk. Spinal cord
4. Cystic fibrosis. compression may occur in exceptional cases.

Histiocytosis X is a benign disorder of unknown aetiology. The diagnosis is based on the demonstration of
It generally affects many systems, but in up to 25% of histiocytes and small round cells in histological specimens of
cases only one system may be involved. The peak age of affected tissues. Management is with steroids, but in some
presentation is between 2 and 4 years, and the condition cases cytotoxic drugs such as vincristine or etoposide may
affects males more than females. The skin, bones, ears, be useful. In some cases spontaneous regression may occur.
lungs, eyes, and the reticuloendothelial and central
nervous systems are most often affected. A skin rash is

Answer 36

1. Amiodarone toxicity. and a KCO estimation. Photosensitivity is common, and
2. Formal lung function tests including a KCO long-term use is associated with slate-grey skin
pigmentation (36a, b). Corneal microdeposits are
estimation. universal and may cause night glare; however, these resolve
on withdrawal of the drug.
The patient has biochemical evidence of hepatitis and
hyperthyroidism. In a patient who has been attending the Side-effects and complications of amiodarone
cardiology clinic for several years, and has underlying atrial
fibrillation, it is highly likely that these abnormalities are Side-effects Complications
due to the toxicity of amiodarone which was prescribed to Nausea Optic neuritis
control the AF. Amiodarone is a class III anti-arrhythmic Metallic taste Peripheral neuropathy
agent which is very effective in the management of Myopathy
ventricular and supraventricular arrhythmias; however, it is in the mouth Hepatitis/cirrhosis
associated with several side-effects and complications Nightmares Alveolitis
(Table). Lung fibrosis is an important complication and Tremor Hyper/hypothyroidism
almost certainly accounts for the patient’s breathlessness. Headaches Epididymitis
Early interstitial fibrosis may not be apparent on chest X- Rashes including Conduction tissue
ray, but is suggested by a low KCO on respiratory function
tests. Both hyperthyroidism and hypothyroidism are phototoxicity disturbances
recognized due to the iodine content of the drug.
Asymptomatic biochemical hepatitis may occur and some
patients may progress to cirrhosis of the liver. Patients on
amiodarone should have an annual thyroid test, an LFT

Clinical Cases 43

Question 37

37a 37b

A 66-year-old retired plumber was seen in the Accident 10 37c
and Emergency Department with progressively increasing 8
dyspnoea and left-sided chest discomfort, which was 6 Expiration
worse on inspiration. He had been seen by his GP one 4 Normal
week previously, and treated with antibiotics for a chest
infection. According to his wife, his appetite had been Flow (l/s) 2 Inspiration
reduced and he had lost at least 6.5 kg in weight over six 0
weeks. He stopped smoking five years previously, but 2 Patient
before this was smoking 30 cigarettes per day for over 40 4
years. 6
8
On examination, he appeared thin and tachypnoeic.
There was no clinical evidence of cyanosis, but he had early 10
clubbing. His voice was hoarse. Respiratory examination
revealed reduced chest expansion of the left hemithorax 0 24 6
and dullness to percussion with reduced air entry affecting Change in lung volume (l)
the left anterior hemithorax. A chest radiograph taken is
shown (37a). Hb 10.8 g/dl
WCC 14.2 ϫ 109/l
The patient was given oral amoxycillin and Platelets 80 ϫ 109/l
erythromycin for one week, and asked to return for MCV 91 fl
repeat chest X-ray in two weeks. However, only four days Sodium 133 mmol/l
later he was admitted as an emergency with increasing Potassium 3.9 mmol/l
confusion. According to his wife, he had been extremely Urea 18 mmol/l
thirsty and was waking several times each night to pass Creatinine 179 μmol/l
large volumes of urine! Glucose 8.1 mmol/l
Urinalysis Glucose 0
Investigations on this occasion are shown. Ketones +
Blood film (37b)
1. What is the probable underlying diagnosis? Flow loop curve (37c)
2. What are the two abnormalities on the flow loop

curve?
3. Give five possible reasons for his confusion.
4. What are the most likely reasons for his polydipsia

and polyuria?
5. Give two reasons why this man is incurable.

44

Answer 37 The chest X-ray (37a) demonstrates a large mass arising
from the left hilum, which is complicated by left upper
1. Bronchial carcinoma affecting the left lung. lobe collapse. The risks for developing bronchial
2. i. Severely reduced expiratory flow consistent with carcinoma are smoking and the possibility of asbestos
exposure during his occupation as a plumber.
obstructive airways disease.
ii. Reduced inspiratory flow limb consistent with The excessive thirst and polyuria reflect diabetes
insipidus. Cerebral metastases may infiltrate the
upper airways obstruction. In this case it is hypothalamus or posterior pituitary to cause cranial
probably due to a laryngeal nerve palsy causing diabetes insipidus by affecting the production and release
vocal cord paralysis. (See ‘Interpretation of of ADH, respectively. Hypercalcaemia may render the
Respiratory Flow Loop Curves’, page 420.) distal renal tubules refractory to the action of ADH and
3. i. Hypercalcaemia. produce a nephrogenic diabetes insipidus. In this case,
ii. Cerebral metastases. the hypercalcaemia is probably due to bony metastases
iii. Hypoxia. because the blood film demonstrates a leuco-
iv. Concurrent respiratory tract infection. erythroblastic picture suggesting marrow infiltration
v. Dehydration. (37b); however, in squamous cell bronchial carcinoma
4. i. Hypercalcaemia. the hypercalcaemia may also be due to ectopic secretion
ii. Cranial diabetes insipidus. of a PTH-related peptide.
5. i. Local and distant metastases (recurrent laryngeal
nerve, bone and brain). The main contraindications to surgery include local
ii. Poor respiratory reserve. and distant metastases and poor respiratory reserve.

Question 38

A 76-year-old Polish male was found collapsed in the toilet 38a
by the home help. He had been generally unwell for the
past five months and was easily fatigued. His appetite was 3 cm below the costal margin. Examination of the central
reduced because he felt constantly nauseous. He was seen nervous system was difficult owing to his confusion and
in the Accident and Emergency Department one month agitation, but there was no evidence of paralysis. He was
ago with abdominal pain, and was diagnosed as having clutching his right forearm and seemed to be in severe pain
constipation, for which he was prescribed senna. He had on the slightest movement of his right arm. The knee and
not been out of bed for four days. He had a past history of ankle jerks were absent, but the plantar response was
hypertension and osteoarthritis. He lived alone and was normal. Examination of the left fundus revealed an
becoming increasingly dependent on the social services as abnormality (38b). Inspection of his legs is shown (38c).
his health deteriorated. He smoked ten cigarettes per day.
Investigations are shown.
On examination, he was confused and agitated, and
appeared pale. He was malnourished and clinically
dehydrated. There was no evidence of cyanosis or
lymphadenopathy. His temperature was 39°C (102.2°F).
The heart rate was 110 beats/min and blood pressure
105/50 mmHg. The JVP was not raised. Examination of
his tongue is shown (38a). On examination of the
cardiovascular system, he had a fourth heart sound.
Respiratory examination revealed a respiratory rate of
30/min. Movement of the right hemithorax was reduced.
Percussion note was dull from the right mid-zone to the
base. The abdomen was thin. A liver edge was palpable

Hb 8 g/dl Clinical Cases 45

WCC 25 ϫ 109/l 38b

Platelets 90 ϫ 109/l 38c
38d
Blood film (38d) 38f

ESR 148 mm/h

Sodium 130 mmol/l

Potassium 4 mmol/l

Urea 16 mmol/l

Total protein 50 g/l

Albumin 18 g/l

Alkaline phosphatase 220 iu/l

AST 26 iu/l

Bilirubin 13 μmol/l

Arterial blood gases (40% O2):
pH 7.4

PaO2 9 kPa
PaCO2 5 kPa
Bicarbonate 18 mmol/l

Chest X-ray (38e)

X-ray, right arm (38f)

Bone marrow aspirate Dry tap

Urinalysis Protein +++

No organisms

1. What is the diagnosis?
2. What is the cause of the abnormality in the left

fundus?
3. Why does the patient have a painful right arm?
4. Comment on two abnormalities seen on his legs

and give an explanation for each one.
5. List five complications of the disease that are

evident in this patient.

38e

46

Answer 38 iii. Bony involvement. There are numerous lytic
lesions affecting the right humerus and a
1. Multiple myeloma. pathological fracture. The blood picture (38d)
2. Branch retinal vein occlusion. reflects a leuco-erythroblastic picture which, in
3. He has a pathological fracture affecting his right this case, is probably due to marrow infiltration
of myeloma cells, although severe sepsis is also a
radius. recognized cause. The typical appearance of
4. i. Oedema due to nephrotic syndrome. plasma cells is shown in the marrow (38g). The
cell nuclei sit at the edge of the cytoplasm and
ii. Bruising secondary to thrombocytopenia or are well defined.
amyloid related capillary fragility.
iv. Hyperviscosity leading to venous thrombosis in
5. i. Sepsis affecting the right lung as a result of the retinal vein. Hyperviscosity occurs when the
immunoparesis. Bronchopneumonia and renal paraprotein level is very high. Venous and arterial
failure are the most common causes of death in thrombosis can occur almost anywhere.
multiple myeloma.
v. Renal failure. The raised urea suggests renal
ii. Amyloidosis. The tongue is enlarged owing to failure. There are several recognized causes of
amyloid infiltration. The patient is oedematous renal failure in myeloma, which are shown
and hypoalbuminaemic and has marked (Table). Myeloma is also a recognized cause of
proteinuria, suggesting nephrotic syndrome, proximal RTA; however, this does not cause
which is a manifestation of renal amyloid. renal failure per se.
Amyloid often affects the heart in myeloma, and
is an important cause of death. Skin deposits, a 38g
peripheral neuropathy and massive hepatic
infiltration are all recognized features of
amyloidosis associated with multiple myeloma.

Recognized causes of renal failure in myeloma

• Tubular obstruction by light chain deposits
• Infiltration of plasma cells in the kidney (‘myeloma kidney’)
• Hypercalcaemia
• Hyperuricaemic nephropathy
• Amyloidosis
• Recurrent pyelonephritis secondary to immunoparesis
• Renal vein thrombosis secondary to hyperviscosity
• Renal failure secondary to analgesics, such as non-steroidal

anti-inflammatory drugs
• Renal failure secondary to contrast agents, e.g. for an IVP

Clinical Cases 47

Question 39 Hb 17 g/dl
WCC 9 ϫ 109/l
A 60-year-old female presented with a four-month history Platelets 300 ϫ 109/l
of headaches and general malaise. Over the past three Sodium 135 mmol/l
weeks she had developed nausea and epigastric pain after Potassium 3.4 mmol/l
meals. She was restless at night and had noticed occasional Urea 10 mmol/l
palpitations, but attributed these symptoms to fear of Creatinine 120 μmol/l
illness. She also had of episodic diarrhoea. On examination, Calcium 2.8 mmol/l
she appeared anxious. She had an irregular swelling in her Phosphate 0.6 mmol/l
neck, which moved vertically with swallowing. Her heart Blood glucose 10 mmol/l
rate was 120 beats/min, and regular; blood pressure was TSH 3 mu/l
180/105 mmHg. Examination of all the major systems
was normal. Investigations are shown.

1. What is the cause of this patient’s epigastric pain?
2. Give two explanations for the diarrhoea.
3. What is the most likely cause for the hypertension?
4. How would you explain her haemoglobin level?
5. What is the complete diagnosis?
6. List two tests you would perform to investigate the

thyroid swelling.

Question 40 40a

M-mode echocardiograms of two
women (A and B), who were
investigated for breathlessness in a
busy cardiology clinic, are shown
(40a, b).

1. What is the diagnosis in female A? 40b
a. Mitral valve prolapse.
b. Hypertrophic cardiomyopathy.
c. Mitral valve endocarditis.
d. Rheumatic mitral stenosis.
e. Aortic regurgitation.

2. What is the diagnosis in female B?
a. Hypertrophic cardiomyopathy.
b. Atrial myxoma.
c. Mitral valve endocarditis.
d. Pericardial effusion.
e. Rheumatic mitral stenosis.

48

Answer 39

1. Hypercalcaemia (peptic ulcer disease, pancreatitis). medulla; the latter is the most common expression of the
2. i. Raised calcitonin levels. disorder. It is rare below the age of 20, and becomes
more common from there onwards. One-third of cases
ii. Phaeochromocytoma. occur in patients aged over 70 years. The genetic
3. Phaeochromocytoma. abnormality is within the RET proto-oncogene on
4. Secondary polycythaemia in association with chromosome 10. The gene encodes a transmembrane
glycoprotein receptor tyrosine kinase.
phaeochromocytoma.
5. Multiple endocrine neoplasia type 2 (MEN 2). Medullary cell carcinoma develops in 80% of cases and
6. i. Fine-needle aspiration of the thyroid. is usually the first presentation; however, MEN 2
accounts for only 10–20% of all medullary cell
ii. Serum calcitonin. carcinomas. Management is with surgery. Persistently
elevated calcitonin levels after surgery suggest that the
The patient has hypercalcaemia and hypertension patient is not cured. Phaeochromocytomas occur in 50%
associated with palpitations. She has an irregular swelling of cases and are usually bilateral and may be
in the neck, which is consistent with a thyroid goitre. The asymptomatic; therefore, annual screening with urinary
TSH is normal, indicating that she is euthyroid. The low catecholamine measurements is advised. Hyperpara-
serum phosphate and high calcium levels indicate thyroidism occurs in only 5–10% and is restricted to
that primary hyperparathyroidism is the most likely certain families.
cause of the raised calcium. Although primary
hyperparathyroidism by itself is a recognized cause of Screening in families with known disease is by genetic
hypertension, it does not account for the palpitations and testing. Patients who are gene-positive should have a
restlessness, which are best explained by a coexisting prophylactic thyroidectomy at a young age, and should
phaeochromocytoma. The combination of primary then be screened annually with serum PTH, and urinary
hyperparathyroidism and phaeochromocytoma should catecholamine estimation. In those families in whom a
lead one to consider the possibility of MEN 2. In the genetic abnormality has not been demonstrated, annual
context of this question it is very probable that the serum calcitonin should also be performed, and fine-
thyroid goitre represents a medullary cell carcinoma. needle aspiration of the thyroid is indicated if a goitre
appears, or if the calcitonin becomes elevated.
MEN 2 consists of tumours arising from thyroid C cell
carcinoma, the parathyroid gland and the adrenal

Answer 40

1. a. Mitral valve prolapse. regurgitation, in whom prophylactic antibiotics during
2. b. Atrial myxoma. dental work are recommended, and supraventricular
arrhythmias. Mitral valve prolapse has been implicated as
Female A (40a): the mitral valve has a normal appearance a cause of sudden cardiac death in highly trained athletes,
in diastole. In systole, both the anterior and posterior but its significance as the only finding at post-mortem is
leaflets appose normally in the beginning; however, in controversial because the high prevalence of the
mid systole there is posterior movement (prolapse) of condition may mean the finding is coincidental, rather
both leaflets, but predominantly the posterior leaflet. than causal. It is more likely that death may have been
This is characteristic of mitral valve prolapse and is caused by the association of mitral valve prolapse with the
responsible for the mid-systolic clicks and/or the mid- Wolff–Parkinson–White syndrome causing atrial
systolic murmur of mitral regurgitation. Mitral valve fibrillation which degenerates to ventricular fibrillation,
prolapse is present in 2–5% of the general population. It or the LQTS, which causes polymorphic ventricular
is generally benign but may be associated with symptoms tachycardia.
of sharp inframammary chest pain, breathlessness even
with normal ventricular function, palpitation and odd Female B (40b): there is echogenic shadowing in the
neurological symptoms. There is a risk of infective mitral valve orifice that is typical of atrial myxoma. The
endocarditis in patients with consequent mitral condition is discussed in Answer 178. (See also
Echocardiography, page 421.)


Click to View FlipBook Version